OB HESI/FINAL, HESI with Rationale 12, HESI OB/MATERNITY Practice Quiz, HESI

Ace your homework & exams now with Quizwiz!

The home health nurse is visiting an older client who was just charge from the hospital 3 days ago following hip pinning surgery. The client lives with her daughter, who prepares the family meals. In discussing nutrition for postoperative healing it, which meal choices should the nurse suggest for this clients diet? (Select all that apply.)`

Answer A. Low-fat milk. B. Oat bran. D. Grilled salmon. E. Baked chicken. Rationale

A male client who was just discharged 3 days ago after an exploratory laparoscopic biopsy is admitted to the hospital with a warm, tender, reddened, and swollen lower left leg. The nurse is preparing to initiate heparin therapy. What additional intervention should the nurse include in this clients plan of care?`

Answer A. Maintain the client on bed rest. Rationale

A client who had a right total knee replacement two days ago is progressed to a soft diet. Which food selections should the nurse recommend to this client? (Select all that apply.)`

Answer A. Pasta with a cream sauce. B. Pancakes with syrup. C. Scrambled eggs and potatoes. D. Steamed rice and cooked squash. Rationale

In caring for the body of a client who just died, which tasks can be delegate to the unlicensed assistive personnel (UAP)? (Select all that apply.)`

Answer A. Place personal religious artifacts on the body. D. Attach identifying name tags to the body. E. Follow cultural beliefs in preparing the body. Rationale

A 56-year-old man shares with the nurse that he is having difficulty making a decision about terminating life support for his wife. What is the best initial action by the nurse?`

Answer A. Provide an opportunity for him to clarify his values related to the decision. Rationale

To evaluate the effectiveness of a male clients new prescription for ezetimibe, what action should the clinic nurse implement?`

Answer A. Remind the client to keep his appointments to have his cholesterol level checked. Rationale

At 1615, prior to ambulating a postoperative client for the first time, the nurse reviews the client's medical record. Based on date contained in the record, what action should the nurse take before assisting the client with ambulation? (Click on each chart tab for additional information. Please be sure to scroll to the bottom right corner of each tab to view all information contained in the client's medical record.)`

Answer A. Remove sequential compression devices. Rationale

During shift report, the central electrocardiogram (EKG) monitoring system alarms. Which client alarm should the nurse investigate firs?`

Answer A. Respiratory apnea of 30 seconds. Rationale

A male client receives a thrombolytic medication following a myocardial infarction. When the client has a bowel movement, what action should the nurse implement?`

Answer A. Send stool sample to the lab for a guaiac test. Rationale

Which assessment finding for a client who is experiencing pontine myelinolysis should the nurse report to the healthcare provider?`

Answer A. Sudden dysphagia. Rationale

The nurse is assisting a new mother with infant feeding. Which information should the nurse provide that is most likely to result in a decreased milk supply for the mother who is breastfeeding?`

Answer A. Supplemental feedings with formula. Rationale

A client is admitted to the mental health unit with relationship distress with spouse and depressed mood. Findings of which diagnostic tests provide the most information for developing this client's plan of care?`

Answer A. Urine drug screen. Rationale

A 26-year-old female client is admitted to the hospital for treatment of a simple goiter, and levothyroxine sodium (Synthroid) is prescribed. Which symptoms indicate to the nurse that the prescribed dosage is too high for this client? The client experiences`

Answer A. palpitations and shortness of breath. Rationale

When delegating a task to an unlicensed assistive personnel (UAP) newly assigned to a nursing unit, what question is most important for the nurse to ask the UAP?`

Answer B. "What experience do you have performing this task?" Rationale

In determining the one minute Apgar score of a male infant, the nurse assesses a heart rate of 120 beats per minute and 44 respirations per minute. He has flaccid muscle tone with slight flexion and slight resistance to straightening. He has a loud cry with stimulation, and his color is acrocyanotic. What is the correct Apgar score for this infant? `

Answer B. 8. Rationale

A client is scheduled to receive an IV dose of ondansetron (Zofran) eight hours after receiving chemotherapy. The client has a saline lock and is sleeping quietly without any restlessness. The nurse caring for the client is not certified in chemotherapy administration. What action should the nurse take?`

Answer B. Administer the Zofran after flushing the saline lock with saline. Rationale

A client diagnosed with calcium kidney stones has a history of gout. A new prescription for aluminum hydroxide (Amphogel) is scheduled to begin at 0730. Which client medication should the nurse bring to the healthcare provider's attention?`

Answer B. Allopurinol (Zyloprim). Rationale

While receiving a male postoperative client's staples the nurse observe that the client's eyes are closed and his face and hands are clenched. The client states, "I just hate having staples removed". After acknowledging the client's anxiety, what action should the nurse implement?`

Answer B. Attempt to distract the client with general conversation. Rational

A 3-year-old boy is brought to the emergency department after the mother found the child in the back yard holding a piece of a toy in his hand and in respiratory distress. The child is dusky with a loud, inspiratory stridor and weak attempts to cough. Which actions should the nurse implement?`

Answer B. Auscultate all pulmonary lung fields and attempt a Heimlich maneuver. Rationale

The nurse is evaluating the diet teaching of a client with hypertension. What dinner selection indicates that the client understands the dietary recommendations for hypertension?`

Answer B. Baked pork chop, applesauce, corn on the cob, 1% milk, and key-lime pie. Rationale

The nurse is assessing a client with diabetes mellitus who is at risk of developing acute renal failure. Which assessment finding is earliest indication of acute renal failure?`

Answer B. Blood urea nitrogen 35 mg/dL (12.4 mmol/L SI units). Rationale

The mother of the 12- month-old with cystic fibrosis reports that her child is experiencing increasing congestion despite the use of chest physical therapy (CPT) twice a day, and has also experiences a loss of appetite. What instruction should the nurse provide?`

Answer B. CPT should be performed more frequently, but at least an hour before meals. Rationale

A client is admitted with the diagnosis of Wernicke's Syndrome. What assessment finding should the nurse use in planning the clients care? `

Answer B. Confusion. Rationale

An older client is admitted with pneumonia, and the healthcare provider prescribes penicillin G potassium IV. Which assessment finding increases the risk of adverse reactions in this client?`

Answer B. Daily use of spironolactone for hypertension. Rationale.

An increased number of elderly persons are electing to undergo a new surgical procedure which cures glaucoma. What effect is the nurse likely to note as a result of this increase in glaucoma surgeries?`

Answer B. Decreased prevalence of glaucoma in the population. Rationale

The nurse is preparing to administer a histamine 2-receptor antagonist to a client with peptic ulcer disease. What is the primary purpose of this drug classification?`

Answer B. Decreases the amount of HCl secretion by the parietal cells in the stomach. Rationale

After a colon resection for colon cancer, a male client is moaning while being transferred to the Postanesthesia Care Unit (PACU). Which intervention should the nurse implement first?`

Answer B. Determine client's pulse, blood pressure, and respirations. Rationale

The nurse is palpating the lymph nodes of an 18-month-old. Which findings should the nurse call to the attention of the healthcare provider?`

Answer B. Enlarged, warm, tender preauricular node. Rationale

A client who received partial thickness (second degree) burns over the anterior surfaces of both arms, legs, and chest in a burning vehicle collision receives a prescription for daily dressing changes and therapeutic baths. The nurse determines that a hoist is required to move the immobile client from a stretcher into the therapeutic bath. Which intervention should the nurse implement first?`

Answer B. Explain the procedure to the client. Rationale

The pathophysiological mechanism are responsible for ascites related to liver failure? (Select all that apply)`

Answer B. Fluid shifts from intravascular to interstitial area due to decreased serum protein. C. Increased hydrostatic pressure in portal circulation increases fluid shifts into abdomen. D. Increased circulating aldosterone levels that increase sodium and water retention. Rationale

The nurse is assisting the mother of a child with phenylketonuria (PKU) to select foods that are in keeping with the child's dietary restrictions. Which foods are contraindicated for this child?`

Answer B. Foods sweetened with aspartame. Rationale

A 60-year-old female client asks the nurse about hormones replacement therapy (HRT) as a means preventing osteoporosis. Which factor in the client's history is a possible contraindication for the use of HRT?`

Answer B. Her mother and sister have a history of breast cancer. Rationale

A group of nurse-managers is asked to engage in a needs assessment for a piece of equipment that will be expensed to the organization's budget. Which question is most important to consider when analyzing the cost-benefit for this piece of equipment?`

Answer B. How many departments can use this equipment? Rationale

Assessment by the home health nurse of an older client who lives alone indicates that client has chronic constipation. The client's fluid and fiber intake is deficient and he eats microwaved foods at home and frequents fast-food restaurants. Daily medications include furosemide for hypertension and heart failure and laxatives. To manage the client's constipation, which suggestions should the nurse provide? (Select all that apply)`

Answer B. Include oatmeal with stewed pruned for breakfast as often as possible. C. Increase fluid intake by keeping water glass next to recliner. D. Recommend seeking help with regular shopping and meal preparation. Rational

An older adult female admitted to the intensive care unit (ICU) with a possible stroke is intubated with ventilator setting of tidal volume 600, PlO2 40%, and respiratory rate of 12 breaths/minute. The arterial blood gas (ABG) results after intubation are pH 7.31. PaCO2 60, PaO2 104, SPO2 98%, HCO3 23. To normalize the client's ABG finding, which action is required?`

Answer B. Increase ventilator rate. Rationale

A woman with an anxiety disorder calls her obstetrician's office and tells the nurse of increased anxiety since the normal vaginal delivery of her son three weeks ago. Since she is breastfeeding, she stopped taking her antianxiety medications, but thinks she may need to start taking them again because of her increased anxiety. What response is best for the nurse to provide this woman?`

Answer B. Inform her that some antianxiety medications are safe to take while breastfeeding. Rationale

A client experiencing withdrawal from the benzodiazepines alprazolam (Xanax) is demonstrating severe agitation and tremors. What is the best initial nursing action?`

Answer B. Initiate seizure precautions. Rationale

The nurse is teaching a client how to perform colostomy irrigations. When observing the client's return demonstration, which action indicates that the client understood the teaching?`

Answer B. Keeps the irrigating container less than 18 inches above the stoma. Rationale

When providing diet teaching for a client with cholecystitis, which types of food choices should the nurse recommend to the client?`

Answer B. Low fat. Rationale

An adult woman who was seen earlier today in the clinic is admitted to the hospital because she is very nervous, has a racing heart beat, and reports a weight loss of 15 pounds in the last month. The healthcare provider suspects that she has hyperthyroidism and prescribes further testing. What intervention should the nurse include in this client's plan of care?`

Answer B. Monitor the client for shortness of breath. Rationale

A client with a history of cirrhosis and alcoholism is admitted with severe dyspnea and ascites. Which assessment finding warrants immediate intervention by the nurse?`

Answer B. Muffled heart sounds. Rationale

An older male adult resident of a long-term care facility is hospitalized for a cardiac catheterization that occurred yesterday. Since the procedure was conducted, the client has become increasingly disoriented. The night shift nurse reports that he attempted to remove the sandbag from his femoral artery multiple times during the night. What actions should the nurse take? (Select all that apply.)`

Answer B. Notify the healthcare provider of the client's change in mental status. C. Include q2 hour reorientation in the client's plan of care. Rationale

The nurse is teaching a male adolescent recently diagnosed with type 1 diabetes mellitus (DM) about self-injecting insulin. Which approach is best for the nurse to use to evaluate the effectiveness of the teaching?`

Answer B. Observe him as he demonstrates the self-technique to another diabetic adolescent. Rationale

An elderly client seems confused and reports the onset of nausea, dysuria, and urgency with incontinence. Which action should the nurse implement?`

Answer B. Obtain a clean catch mid-stream specimen. Rationale

In early septic shock states, what is the primary cause of hypotension?`

Answer B. Peripheral vasodilation. Rationale

A client with diabetic peripheral neuropathy has been taking pregabalin (Lyrica) for 4 days. Which finding indicates to the nurse that the medication is effective?`

Answer B. Reduced level of pain. Rationale

A female client's estranged husband arrives at the hospital and demands that his wife have no other visitors. The client becomes angry and insists that the estranged husband be barred from visiting her. Which intervention should the nurse implement?`

Answer B. Request a multidisciplinary care conference to discuss husband's demands. Rationale

While caring for a client with a cervical spine injury, which assessment finding should the nurse report to the healthcare provider immediately?`

Answer B. Respiratory rate 6 breaths/minute. Rationale

While caring for a client who is mechanically ventilated, the nurse response to a high-pressure alarm. Which assessment finding warrants immediate intervention by the nurse?`

Answer B. Restless client who is biting the endotracheal tube. Rationale

To reduce staff nurse role ambiguity, which strategy should the nurse-manager implement?`

Answer B. Review the staff nurse job description to ensure that it is clear, accurate, and recurrent. Rationale

A client refuses to ambulate, reporting abdominal discomfort and bloating caused by "too much gas buildup". The client's abdomen is distended. Which prescribed PRN medication should the nurse administer?`

Answer B. Simethicone (Mylicon). Rationale

The nurse is caring for a group of clients with the help of a licensed practical nurse (LPN) and an experienced unlicensed assistive personnel (UAP). Which procedures can the nurse delegate to the UAP? (Select all that apply.)`

Answer B. Take postoperative vital signs for a client who has an epidual following knee arthroplasty. D. Collect a sputum specimen for a client with a fever of unknown origin E. Ambulate a client who had a femoral-popliteal bypass graft yesterday. Rationale

A client who recently underwear a tracheostomy is being prepared for discharge to home. Which instructions is most important for the nurse to include in the discharge plan?`

Answer B. Teach tracheal suctioning techniques. Rationale

An elderly client with degenerative joint disease asks if she should use the rubber jar openers that are available. The nurse's response should be based on which information about assistive devices?`

Answer B. They decrease the risk for joint trauma. Rationale

When caring for a client who has acute respiratory distress syndrome (ARDS), the nurse elevates the head of the bed 30 degrees. What is the reason for this intervention?`

Answer B. To reduce abdominal pressure on the diaphragm. Rationale

When conducting diet teaching for a client who was diagnosed with hypoparathyroidism, which foods should the nurse encourage the client to eat? (Select all that apply.)`

Answer B. Yogurt. E. Processed cheese. Rationale

The nurse is assigned to care for clients on a medical unit. Based on the notes taken during the shift report, which client situation warrants the nurse's immediate attention?`

Answer C. A 10-year-old who is receiving chemotherapy and the infusion pump is beeping. Rationale

A primigravida client at 36-weeks gestation is admitted to labor and delivery unit because her membranes ruptured 30 minutes ago. Initial assessment indicates 2 cm cervical dilation, 50% effaced, -2 station, vertex presentation, greenish colored amniotic fluid, and contractions occurring 3 to 5 minutes with a decrease in fetal heart rate after the last contraction peaks. Which action should the nurse implement first?`

Answer C. Administer Oxygen via face mask. Rationale

While changing a client's postoperative dressing, the nurse observes a red and swollen wound with a moderate amount of yellow and green drainage and a foul odor. Given there is a positive methicillin-resistant Staphylococcus aureus (MRSA), which is the most important action for the nurse to take?`

Answer C. Administer prescribed antibiotics. Rationale

The healthcare provider changes a client's medication prescription from IV to PO administration and doubles the dose. The nurse notes in the drug guide that the prescribed medication, when given orally, has a high first-pass effect and reduced bioavailability. What action should the nurse implement?`

Answer C. Administer the medication via the oral route as prescribed. Rationale

A woman at 24 weeks gestation who has fever, bodyaches, and has been coughing for the last 5 days is sent to the hospital with admission prescriptions for H1N1 influenza. Which prescription has the highest priority?`

Answer C. Assign private room. Rationale

A male client has received a prescription for orlistat for weight and nutrition management. In addition to the medication, the client states he plans to take a multivitamin. What teaching should the nurse provide?`

Answer C. Be sure to take the multivitamin and the medication at least two hours apart for best absorption and effectiveness. Rationale

A female client is admitted for diabetic crisis resulting from inadequate dietary practices. After stabilization, the nurse talks to the client about her prescribed diet. What client characteristic is most import for successful adherence to the diabetic diet?`

Answer C. Demonstrates willingness to adhere to the diet consistently. Rationale

In preparing a diabetes education program, which goal should the nurse identify as the primary emphasis for a class on diabetes self-management?`

Answer C. Enable clients to become active participating in controlling the disease process. Rationale

A male client's laboratory results include a platelet count of 105,000/mm3. Based on this finding, the nurse should include which action in the client's plan of care?`

Answer C. Encourage him to use an electric razor. Rationale

A newly hired male unlicensed assistive personnel (UAP) is assigned to a home healthcare team along with two experienced UAPs. Which intervention should the home health nurse implement to ensure adequate care for all clients?`

Answer C. Evaluate the newly hired UAP's level of competency by observing him deliver care. Rationale

A client has been taking an oral corticosteroid for two weeks. Nursing assessment reveals that the client has developed a rounded face. What action should the nurse take in response to this finding?`

Answer C. Explain this side effect to the client. Rationale

When development a teaching plan for a client with newly diagnosed Type 1 diabetes, the nurse should explain that an increased thirst is an early sing of diabetes ketoacidosis (DKA). Which action should the nurse instruct the client to implement if this sign of DKA occurs ?`

Answer C. Give a dose of regular insulin per sliding scale. Rationale

The nurse receives a newborn within the first minutes after a vaginal delivery and intervenes to establish adequate respirations. What priority issue should the nurse address to ensure the newborn's survival?`

Answer C. Heat loss. Rationale

The healthcare provider prescribes acarbose (Precose), an alpha-glucosidase inhibitor, for a client with Type 2 diabetes mellitus. Which information provides the best indicator of the drug's effectiveness?`

Answer C. Hemoglobin A1C (HbA1C) reading less than 7%. Rationale

A client with cirrhosis is receiving a low protein diet. The nurse should explain to the family that the diet restriction is implemented to reduce the risk of which complication of cirrhosis?`

Answer C. Hepatic encephalopathy. Rationale

The nurse learns during shift report that a client is experiencing frequent ectopic beats on the cardiac telemetry monitor. Which assessment findings should the nurse expect this client to exhibit?`

Answer C. Irregular heart rhythm. Rationale

After placing a stethoscope as seen in the picture, the nurse auscultates S1 and S2 heart sounds. To determine if an S3 heart sound is present, what action should the nurse take next?`

Answer C. Listen with the bell at the same location. Rationale

A 66-year-old woman is retiring and will no longer have a health insurance through her place of employment. Which agency should the client be referred to by the employee health nurse for health insurance needs?`

Answer C. Medicare. Rationale

The nurse is auscultating a client's heart sounds. Which description should the nurse use to document this sound? (Please listen to the audio first to select the option that applies.)`

Answer C. Murmur. Rationale

A mother brings her 3-year-old son to the emergency room and tells the nurse the he has had an upper respiratory infection for the past two days. Assessment of the child reveals a rectal temperature of 102° F. He is drooling and becoming increasingly more restless. What action should the nurse take first?`

Answer C. Notify the healthcare provider and obtain a tracheostomy tray. Rationale

A female client with possible acute renal failure (ARF) is admitted to the hospital and mannitol (Osmitrol) is prescribed as a fluid challenge. Prior to carrying out this prescription, what intervention should the nurse implement? `

Answer C. Obtain vital signs and breath sounds. Rationale

A client with rheumatoid arthritis (RA) starts a new prescription of etanercept (Enbrel) subcutaneously once weekly. The nurse should emphasize the importance of reporting which problem to the healthcare provider?`

Answer C. Persistent fever. Rationale

Two clients ring their call bells simultaneously requesting pain medication. What action should the nurse implement first?`

Answer C. Provide a family tour of the preoperative unit one week before the surgery is scheduled. Rationale

When assessing and adult male who presents as the community health clinic with a history of hypertension, the nurse note that he has 2+ pitting edema in both ankles. He also has a history of gastroesophageal reflex disease (GERD) and depression. Which intervention is the most important for the nurse to implement?`

Answer C. Review the client's use of over the counter (OTC) medications. Rationale

Following discharge teaching, a male client with a duodenal ulcer tells the nurse the he will drink plenty of dairy products, such as milk, to help coat and protect his ulcer. What is the best follow-up action by the nurse?`

Answer C. Review with the client the need to avoid foods that are rich in milk and cream. Rationale

The nurse identifies the presence of a clear fluid on the surgical dressing of a client who just returned to the unit following lumbar spinal surgery. What action should the nurse implement immediately?`

Answer C. Test the fluid on the dressing for glucose using a chemstrip. Rationale

A client who receives multiple antihypertensive medications experiences syncope due to a drop in blood pressure to 70/40. What is the rationale for the nurse's decision to hold the client's scheduled antihypertensive medications?`

Answer C. The additive effect of multiple medications has caused the blood pressure to drop too low. Rationale

The nurse is assessing an older adult with Type 2 diabetes mellitus. Which assessment finding indicates that the client understands long- term control of diabetes?`

Answer C. The hemoglobin A1C was 6.5g/100 ml last week. Rationale

What explanation is best for the nurse to provide a client who asks the purpose of using the log-rolling technique for turning?`

Answer C. The technique is intended to maintain straight spinal alignment. Rationale

A male client is discharged from the intensive care unit following a myocardial infarction, and the healthcare provider prescribes a low-sodium diet. Which lunch selection indicates to the nurse that this client understands the dietary restrictions?`

Answer C. Turkey salad sandwich. Rationale

A female client reports that her hair is becoming coarse and breaking off, that the outer part of her eyebrows have disappeared, and that her eyes are all puffy. Which follow-up question is best for the nurse to ask?`

Answer D. "Have you noticed any changes in your fingernails?" Rationale

Following a lumbar puncture, a client voices several complaints. What complaint indicates to the nurse that the client is experiencing a complication?`

Answer D. "I have a headache that gets worse when I sit up." Rationale

The nurse is conducting intake interviews of children at a city clinic. Which child is most susceptible to contracting lead poisoning? `

Answer D. A 2-year-old who plays on aging outdoor playground equipment. Rationale

The nurse working in the psychiatric clinic has phone messages from several clients. Which call should the nurse return first?`

Answer D. A family member of a client with dementia who has been missing for five hours. Rationale

A client with a history of chronic pain requests a nonopioid analgesic. The client is alert but has difficulty describing the exact nature and location of the pain to the nurse. Which action should the nurse implement next?`

Answer D. Administer the analgesic as requested. Rationale

Following surgery, a male client with antisocial personality disorder frequently requests that a specific nurse be assigned to his care and is belligerent when another nurse is assigned. What action should the charge nurse implement?`

Answer D. Advise the client that assignments are not based on clients requests. Rationale

While the nurse is providing morning care for a client with chronic obstructive pulmonary disease (COPD), the client becomes very dyspneic and starts to panic. What action should the nurse implement first? `

Answer D. Assist the client to an upright position. Rationale

A male client, who is 24 hours postoperative for an exploratory laparotomy, complains that he is "starving" because he has had no "real food" since before the surgery. Prior to advancing his diet, which intervention should the nurse implement?`

Answer D. Auscultate bowel sounds in all four quadrants. Rationale

The nurse identifies an electrolyte imbalance, an elevated pulse rate, and an elevated blood pressure for a client with chronic kidney disease. Which is the most important action for the nurse to take?`

Answer D. Auscultate for irregular heart rate. Rationale

A male client with angina pectoris is being discharged from the hospital. What instruction should the nurse plan to include in this discharge teaching?`

Answer D. Avoid all isometric exercises, but walk regularly. Rationale

Which breakfast selection indicates that the client understands the nurse's instructions about the dietary management of osteoporosis?`

Answer D. Bagel with jelly and skim milk. Rationale

A 13-year-old client with non-union of a comminuted fracture of the tibia is admitted with osteomyelitis. The healthcare provider collects bone aspirate specimens for culture and sensitivity and applies a cast to the adolescent's lower leg. What action should the nurse implement next?`

Answer D. Begin parenteral antibiotic therapy. Rationale

A client is receiving mesalamine 800 mg PO TID. Which assessment is most important for the nurse to perform to assess the effectiveness of the medication?`

Answer D. Bowel patterns. Rationale

An older male client with Type 2 diabetes mellitus reports that has experiences leg pain when walking short distances, and that the pain is relieved by rest. Which client behavior indicates an understanding of healthcare teaching to promote more effective arterial circulation?`

Answer D. Completely stop cigarette/ cigar smoking. Rationale

A nurse is conducting a physical assessment of a young adult. Which information provides the best indication of the individual nutritional status?`

Answer D. Condition of hair, nails, and skin. Rationale

The nurse assesses a client who has just returned from a diagnostic study, as seen in the picture. The client has a prescription for a nasogastric tube to low intermittent suction and now reports feelings of nausea. What action should the nurse implement first?`

Answer D. Connect the tube to suction. Rationale

A client with hyperthyroidism is being treated with radioactive iodine (I-131). Which explanation should be included in preparing this client for this treatment?`

Answer D. Describe radioactive iodine as a tasteless, colorless medication administered by the healthcare provider. Rationale

Before preparing a client for the first surgical case of the day, a part-time scrub nurse asks the circulating nurse if a 3 minute surgical hand scrub is adequate preparation for this client. Which response should the circulating nurse provide?`

Answer D. Direct the nurse to continue the surgical hand scrub for a 5 minute duration. Rationale

In assessing an adult client with a partial rebreather mask, the nurse notes that the oxygen reservoir bag does not deflate completely during inspiration and the client's respiratory rate is 14 breaths / minute. What action should the nurse implement?`

Answer D. Document the assessment data. Rationale

An adult female client tells the nurse that though she is afraid her abusive boyfriend might one day kill her, she keeps hoping that he will change. What action should the nurse take first?`

Answer D. Explore client's readiness to discuss the situation. Rationale

A female client with otosclerosis is scheduled for a stapedectomy. What information is most important to provide the client about the postoperative care?`

Answer D. Hearing may seem muffled initially. Rationale

The nurse walks into a client's room and notices bright red blood on the sheets and on the floor by the IV pole. Which action should the nurse take first?`

Answer D. Identify the source and amount of bleeding. Rationale

A nurse assesses a client whose hand begins to spasm when the blood pressure cuff is inflated. The client complains of parenthesia in the fingers and toes. Which serum laboratory findings should the nurse expect to find?`

Answer D. Low serum calcium. Rationale

While assisting a client who recently had a hip replacement onto the bed pan, the nurse notices that there is a small amount of bloody drainage on the surgical dressing, the client's skin is warm to the touch, and there is a strong odor from the urine. Which action should the nurse take?`

Answer D. Measure the client's oral temperature. Rationale

A client with cervical cancer is hospitalized for insertion of a sealed internal cervical radiation implant. While providing care, the nurse finds the radiation implant in the bed. What action should the nurse take?`

Answer D. Place the implant in a lead container using long-handled forceps. Rationale

While administering a continuous insulin infusion to a client with diabetic ketoacidosis, it is essential for the nurse to monitor which serum lab value?`

Answer D. Potassium. Rationale

What action should the nurse take first when discontinuing and indwelling urinary catheter?`

Answer D. Remove the normal saline from the balloon. Rationale

A 6-month-old is admitted to the hospital with diarrhea. The mother is feeding the infant a bottle of tap water and tells the nurse that the baby has taken three 8-ounce bottles of water in the last four hours. Which laboratory finding is most important for the nurse to monitor?`

Answer D. Serum sodium levels. Rationale

When assessing a 6-month old infant, the nurse determines that the anterior fontanel is bulging. In which situation would this finding be most significant?`

Answer D. Sitting upright. Rationale

A nurse-manager is preparing the curricula for a class for charge nurses. A staffing formula based on what data ensures quality client care and is most cost-effective?`

Answer D. Skills of staff and client acuity. Rationale

An older male client with a history of Type 1 diabetes has not felt well the past few days and arrives at the clinic with abdominal cramping and vomiting. He is lethargic, moderately, confused, and cannot remember when he took his last dose of insulin or ate last. What action should the nurse implement first?`

Answer D. Start an intravenous (IV) infusion of normal saline. Rationale

A client who is taking an oral dose of a tetracycline complains of gastrointestinal upset. What snack should the nurse instruct the client to take with the tetracycline?`

Answer D. Toasted wheat bread and jelly. Rationale

A client with history of bilateral adrenalectomy is admitted with a weak, irregular pulse, and hypotension. Which assessment finding warrants immediate intervention by the nurse?`

Answer D. Ventricular arrhythmias. Rationale

A client with a history of a bilateral adrenalectomy is admitted with a weak, irregular pulses, and hypotension. Which assessment finding warrants immediate intervention by the nurse?`

Answer D. Ventricular arrhythmias. Rationale

During a postpartum assessment of a client who is five hours post vaginal delivery, the nurse determines the fundus is three finger breadths above the umbilicus and positioned to the client's left side. What action should the nurse implement first?`

Answer A. Encourage the client to void. Rationale

The nurse plans to collect a 24-hour urine specimen for a creatinine clearance test. Which instruction should the nurse provide to the adult male client?`

Answer B. Urinate at specified time, discard the urine, and collect all subsequent urine during the next 24 hours. Rationale

The nurse is planning the preoperative teaching plan for a 12-year-old child who is scheduled for surgery. To help reduce this child's anxiety, which action is the best for the nurse to implement?`

Answer C. Provide a family tour of the preoperative unit one week before the surgery is scheduled. Rationale

A male client with hypertension, who received new antihypertensive prescriptions at his last visit returns to the clinic two weeks later to evaluate his blood pressure (BP). His BP is 158/106 and he admits that he has not been taking the prescribed medication because the drugs make him "feel bad". In explaining the need for hypertension control, the nurse should stress that an elevated BP places the client at risk for which pathophysiological condition?`

Answer C. Stroke secondary to hemorrhage. Rationale

When performing postural drainage on a client with Chronic Obstructive Pulmonary Disease (COPD), which approach should the nurse use?`

Answer D. Explain that the client may be placed in five positions. Rationale

When entering a client's room, the nurse discovers that the client is unresponsive and pulseless. The nurse initiate CPR and Calls for assistance. Which action should the nurse take next?`

Answer D. Place cardiac monitor leads on the client's chest. Rationale

Which action should the nurse implement with auscultating anterior breath sounds? (Place the first action on top and last action on the bottom.)`

Answer Correct order: (PADD) 1. Place stethoscope in suprasternal area to auscultate for bronchial sounds 2. Auscultate bronchovesicular sounds from side to side of the first and second intercostal spaces. 3. Displace female breast tissue and apply stethoscope directly on chest wall to hear vesicular sounds. 4. Document normal breath sounds and location of adventitious breath sounds. Rationale

A woman who gave birth 48 hours ago is bottle feeding her infant. During assessment, the nurse determines that both breasts are swollen, warm, and tender upon palpation. What action should the nurse take?

Apply cold compress to both breast for comfort

A 40-week gestation primigravida client is being induced with an oxytocin (Pitocin) secondary infusion and complains of pain in her lower back. Which intervention should the nurse implement?

Apply firm pressure to sacral area

The nurse is assessing a client who is having a non-stress test at 41 weeks gestation. The nurse determines that the client is not having contractions, the fetal heart rate baseline is 144 bpm, and no FHR accelerations are occurring. What action should the nurse take?

Ask the client if she has felt any fetal movement

The nurse is assessing a client who is having a non-stress test (NST) at 41-weeks gestation. the nurse determines that the client is not having contractions, the fetal heart rate baseline is 144bpm, and no FHR accelerations are occurring. What action should the nurse take?

Ask the client if she has felt any movement.

A client in active labor is becoming increasingly fearful because her contractions are occurring more often than she had expected. Her partner is also becoming anxious. Which of the following should be the focus of the nurse's response?

Asking the client and her partner if they would like the nurse to stay in the room

When assessing a client at 12 weeks of gestation, the nurse recommends that she and her husband consider attending childbirth preparation classes. When is the best time for the couple to attend these classes?

At 30 weeks of gestation

The nurse is teaching a new mother about diet and breastfeeding. Which instruction is most important to include in the teaching plan?

Avoid alcohol because it is excreted in breast milk.

The nurse in a health care clinic is instructing a pregnant client how to perform "kick counts." Which statement by the client indicates a need for further instructions?`

B. "I need to lie flat on my back to perform the kick count"

A pregnant patient asks the nurse Kate if she can take castor oil for her constipation. How should the nurse respond?`

B. "No, it can initiate premature uterine contractions."

During a lecture on reproduction, a student nurse asks the instructor what determines the sex of a fetus. Accurate information in response to this question would be:`

B. "The fertilization of the zygote is the point at which sex is determined."

The nurse is performing an assessment of a pregnant client who is at 28 weeks of gestation. The nurse measures the fundal height in centimeters and expects which finding?`

B. 30 cm

Which growth and development characteristic should the nurse consider when monitoring the effects of a topical medication for an infant?`

B. A thin stratum corneum that increases topical absorption.

The nurse has completed the diet teaching of a male client who is being discharged following treatment of a leg wound. A high protein diet is encouraged to promote wound healing. Which lunch choice by the client indicates that the teaching was effective? `

B. A tunafish sandwich with chips and ice cream.

What intervention is required for a teenager experiencing acute glomerulonephritis with a BP of 170/88 (previously 210/110)?`

B. Administer PRN nifedipine (Procardia) sublingually

Which interventions should the nurse include in the teaching plan for the mother of a 6-year-old who is experiencing encopresis secondary to a fecal impaction? (Select all that apply.)`

B. Administer mineral oil daily. D. Eliminate dairy products. E. Initiate consistent toileting routine.

A 6-month-old boy and his mother are at the health clinic for a well-baby checkup and routine immunization. The HP recommends an influenza vaccine. What medications should the RN plan to administer?`

B. All the immunizations with the influenza being administered in a different site

The nurse caring for a laboring client encourages her to void at least q2h, and records each time the client empties her bladder. What is the primary reason for implementing this nursing intervention?`

B. An over-distended bladder could be traumatized during labor as well as prolong the progress of labor.

The nurse caring for a laboring client encourages her to void at least q2h, and records each time the client empties her bladder. What is the primary reason for implementing this nursing intervention?`

B. An over-distended bladder could be traumatized during labor, as well as prolong the progress of labor

A child with sickle cell anemia who is in vaso-occlusive crisis is admitted to the hospital. Which health care provider prescription would assist in reversing the vaso-occlusive crisis?`

B. Begin intravenous fluids

The nurse is developing a teaching plan for a patient who is 8 weeks pregnant. The LPN should tell the patient that she can expect to feel the fetus move at which time?`

B. Between 16 and 20 weeks' gestation

The nurse is caring for an infant with a diagnosis of bladder exstrophy. To protect the exposed bladder tissue, the nurse should plan which intervention?`

B. Cover the bladder with a nonadhering plastic wrap.

A child is admitted to the pediatric unit with a diagnosis of acute stage Kawasaki disease. In performing an assessment on the child, which findings are characteristic of this disorder? Select all that apply.`

B. Cracking lips C. Conjunctival hyperemia D. Desquamation of the skin

A 3-year-old boy is brought to the emergency room after swallowing an entire bottle of multi-vitamins. Which intervention should RN implement first?`

B. Determine the child's pulse and respirations

The mother of a preschool-aged child asks the nurse if it is all right to administer Pepto Bismol to her son when he "has a tummy ache." After reminding the mother to check the label of all over-the-counter drugs for the presence of aspirin, which instruction should the nurse include when replying to this mother's question?`

B. Do not give if the child has chickenpox, flu, or any other viral illnesses

The nurse is teaching the parents of a 2-year-old child with a congenital heart defect about signs and symptoms of congestive heart failure. Which information about the child is most important for the parents to report to the health care provider?`

B. Exhibits a sudden and unexplained weight gain

A 28-year-old client in active labor complains of cramps in her leg. What intervention should the nurse implement? `

B. Extend the leg and dorsiflex the foot

A client in active labor is admitted with preeclampsia. Which assessment finding is most significant in planning this client's care?`

B. Extend the leg and dorsiflex the foot

28 year old client in active labor complains of cramps in her leg.What intervention should the nurse implement. `

B. Extend the leg and dorsiflex the foot.

During discharge teaching of a child with juvenile rheumatoid arthritis, the nurse should stress to the parents the importance of obtaining which diagnostic testing?`

B. Eye exams

Q.12) A nurse is collecting data during the admission asessment of a client who is pregnant with twins. The client also has 5 year old child. The nurse would document which gravida and para status on this client?`

B. G2P1

The nurse in the gynecology clinic is reviewing the record of a pregnant client after the first prenatal visit. The nurse notes that the health care provider has documented that the woman has a platypelloid pelvis. On the basis of this documentation, the nurse plans care, knowing that this type of pelvis has which characteristic?`

B. Has a flat

Which maternal behavior is the nurse most likely to see when a new mother receives her infant for the first time?`

B. Her arms and hands receive the infant and she then traces the infant's profile with her fingertips

A primigravida, when returning for the results of her multiple marker screening (triple screen), asks the nurse how problems with her baby can be detected by the test. What information will the nurse give to the client to describe best how the test is interpreted?`

B. If MSAFP and estriol levels are low and the hCG level is high, results are positive for a possible chromosomal defect.

A full term infant is transferred to the nursery from labor and delivery. Which information is most important for the nurse to receive when planning immediate care for the newborn? `

B. Infant's condition at birth and treatment received.

client who is attending antepartum classes asks the nurse why her healthcare provider has prescribed iron tablets. The nurse's response is based on what knowledge? `

B. It is difficult to consume 18 mg of additional iron by diet alone.

When inserting a nasogastric tube into the stomach of a 3-month-old infant, which nursing intervention is most important to implement?`

B. Monitor the infant's heart rate.

Which of the following would the nurse identify as a presumptive sign of pregnancy? `

B. Nausea and vomiting

An infant is receiving digoxin for congestive heart failure. The apical heart rate is assessed at 80 beats/min. What intervention should the nurse implement?`

B. Obtain a therapeutic drug level.

A nurse performs a vaginal assessment on a pregnant client in labor. On assessment, the nurse notes the presence of the umbilical cord protruding from the vagina. Which is the initial nursing action?`

B. Place the client in Trendelenburg's position.

The health care provider prescribes patching for a child with strabismus of the right eye, and the nurse instructs the mother regarding this procedure. What should the nurse include in the instructions?`

B. Place the patch on the left eye.

A six month old returns from surgery with elbow restraints. What nursing care should be included when caring for a child in restraints?`

B. Remove restraints one at a time and provide ROM exercises

Which of the following prenatal laboratory test values would the nurse consider as significant?`

B. Rubella titer less than 1:8

The nurse should explain to a 30-year-old gravid client that alpha fetoprotein testing is recommended for which purpose?`

B. Screen for neural tube defects

A 3-year-old client with sickle cell anemia is admitted to the Emergency Department with abdominal pain. The nurse palpates an enlarged liver, an x-ray reveals an enlarged spleen, and a CBC reveals anemia. These findings indicate which type of crisis?`

B. Sequestration.

An 18-month-old child returns to the unit following a cardiac catheterization with a cannulated femoral artery site. Which intervention should the nurse implement?`

B. Show the parents how to hold the child with the extremity extended.

A nurse receives a shift change report for a newborn who is 12 hours post-vaginal delivery. In developing a plan of care, the nurse should give the highest priority to which finding?`

B. Skin color that is slightly jaundiced

The vital signs of 4 yr old child w/ polyuria are: BP 80/40, pulse, 118, and Resp. 24. The child's pedal pulses are present w/ a volume of +1, and no edema is observed. What action should the nurse implement first?`

B. Start an IV infusion of normal saline

Which class of antiinfective drugs is contraindicated for use in children under 8 yrs of age?`

B. Teteracyclines

A client at 36 weeks' gestation is schedule for a routine ultrasound prior to an amniocentesis. After teaching the client about the purpose for the ultrasound, which of the following client statements would indicate to the nurse in charge that the client needs further instruction?`

B. The ultrasound identifies blood flow through the umbilical cord

Which action by the nurse is most helpful in communicating with a preschool-aged child?`

B. Use a doll to play and communicate

A client with gestational htn is an active labor and receiving an infusion of magnesium sulfate. Which drug should the nurse available for signs of potential toxicity?`

B. calcium gluconate

During which of the following would the focus of classes be mainly on physiologic changes, fetal development, sexuality, during pregnancy, and nutrition? `

B. first trimester

A 25-year-old client with diabetes type I visits the clinic to discuss her and her husband's desire to start a family. This diabetic client `

B. has a greater risk of complications during pregnancy

30-year-old gravida 2, para 1 client is admitted to the hospital at 26-weeks' gestation in preterm labor. She is given a dose of terbutaline sulfate (Brethine) 0.25 mg subcutaneous. Which assessment is the highest priority for the nurse to monitor during the administration of this drug? `

B.Maternal and fetal heart rates.

A new mother asks the nurse, "How do I know that my daughter is getting enough breast milk?" Which explanation should the nurse provide? `

B.Your milk is sufficient if the baby is voiding pale straw-colored urine 6 to 10 times a day.

The nurse should explain to a 30 year old gravid client that alpha fetoprotein testing is recommended for which purpose?`

B.screen for neural tube defects

A vaginally delivered infant of an HIV positive mother is admitted to the newborn nursery. What intervention should the nurse perform first?

Bathe the infant with an antimicrobial soap

The nurse is teaching a woman how to use her basal body temperature (BBT) pattern as a tool to assist her in conceiving a child. Which temperature pattern indicates the occurrence of ovulation, and therefore, the best time for intercourse to ensure conception?

Between the time the temperature falls and rises

A multigravida client at 41-weeks gestation presents in the labor and delivery unit afer a non-stress test indicatied that the fetus is experiencing some difficulties in utero. Which diagnostic test should the nurse prepare the client for additional information about fetal status?

Biophysical profile (BPP)

A client who is 3 days postpartum and breastfeeding asks the nurse how to reduce breast engorgement. Which instruction should the nurse provide?

Breastfeed the infant every 2 hours.

A breastfeeding postpartum client is diagnosed with mastitis, and antibiotic therapy is prescribed. Which instruction should the nurse provide to this client?

Breastfeed the infant, ensuring that both breasts are completely emptied.

The nurse has provided instructions about measures to clean the penis to a mother of a male newborn who is not circumcised. Which statement, if made by the mother, indicates an understanding of how to clean the newborn's penis?`

C. "I need to avoid pulling back the foreskin to clean the penis because this may cause adhesions."

Several children have contracted rubeola (measles) in a local school, and the school nurse conducts a teaching session for the parents of the schoolchildren. Which statement made by a parent indicates a need for further teaching regarding this communicable disease?`

C. "The disease can be spread to others 10 days before any sign of the disease appears to 15 days after the rash appears."

A pregnant client at 10 weeks' gestation calls the prenatal clinic to report a recent exposure to a child with rubella. The nurse reviews the client's chart. What is the nurse's best response to the client? `

C. "You were wise to call. Your rubella titer indicates that you are immune and your baby is not at risk."

The nurse is collecting data from a client during the first prenatal visit. The client is anxious to know the gender of the fetus and asks the nurse when she will be able to know. The nurse should respond to the client knowing that the gender of the fetus is determined by which weeks?`

C. 13 to 16

Which of the following represents the average amount of weight gained during pregnancy?`

C. 25 to 35 lbs

The nurse is using the Silverman-Anderson index to assess an infant with respiratory distress and determines that the infant is demonstrating marked nasal flaring, an audible expiratory grunt, and just visible intercostal and xiphoid retractions. Using this scale, which score should the nurse assign?`

C. 5

A 3-month-old infant weighing 10 lb 15 oz has an axillary temperature of 98.9° F. What caloric amount does this child need?`

C. 600 calories/day

The LPN is preparing to administer Solu-medrol 40 mg mixed in 150 mL of sodium chloride via intravenous piggyback. The medication is to be administered over 30 minutes. Using the tubing with a drop factor of 15 ggts/mL, what would the LPN calculate the rate to be in drops per minute?`

C. 75

A client at 32-weeks gestation is hospitalized with severe pregnancy-induced hypertension (PIH), and magnesium sulfate is prescribed to control the symptoms. Which assessment finding indicates the therapeutic drug level has been achieved?`

C. A decrease in RR from 24 to 16

A client with severe preeclampsia is admitted to the maternity department. Which room assignment would be most appropriate for this client?`

C. A private room two doors away from the nurses' station

Surgery is being delayed for an infant with undescended testes. In collaboration with the healthcare provider and the family, which prescription should the nurse anticipate.`

C. A trial of chorionic gonadotrophic hormone

A 16-year old boy is brought to the E.D. with a crushed leg after falling off a horse. His last tetanus toxoid booster was received at 8-years old. Which action should the nurse take?`

C. Administer tetanus toxoid booster

The nurse should teach the parents of a child with a cyanotic heart defect to perform which action when a hypercyanotic spell occurs?`

C. Allow the child to assume a knee-chest position, with the head and chest slightly elevated.

Prior to discharge, what instructions should the nurse give to parents regarding the newborn's umbilical cord care at home?`

C. Allow the cord to air-dry as much as possible.

A client LMP began July 5. Her EDD should be which of the following? `

C. April 12

An adolescent with diabetes receives 30 units of Humulin N insulin at 7:00 am. In accordance with the peak insulin action time, the nurse would monitor for a hypoglycemic episode at what time?`

C. Before supper

The parents of a child with a cleft lip are concerned and ask the nurse when the lip will be repaired. With which statement should the nurse respond?`

C. Cleft-lip repair is usually performed during the first weeks of life.

A preschool-age child who is hospitalized for hypospadias repair is most strongly influenced by which behavior?`

C. Concerns for body integrity

The nurse in the newborn nursery is preparing to complete an initial assessment on a newborn infant who was just admitted to the nursery. The nurse should place a warm blanket on the examining table to prevent heat loss in the infant caused by which method?`

C. Conduction

The nurse observes a 4-year-old boy in a day care setting. Which behavior should the nurse consider normal for this child?`

C. Demonstrates aggressiveness by boasting when telling a story

Which behavior would the nurse expect a 2-year-old to exhibit?`

C. Display possessiveness of toys

The nurse identifies substance abuse behaviors exhibited by a pregnant client during an initial prenatal screening. While promoting a therapeutic and accepting environment, the care managment by the nurse would be MOST appropriate if focused on which of the following?`

C. Eliminating substance use during pregnancy

The mother of a 6-year-old child who has type 1 diabetes mellitus calls a clinic nurse and tells the nurse that the child has been sick. The mother reports that she checked the child's urine and it was positive for ketones. The nurse should instruct the mother to take which action?`

C. Encourage the child to drink liquids.

A new mother is having trouble breastfeeding her newborn. The child is making frantic rooting motions and will not grasp the nipple. Which intervention should the nurse implement?`

C. Encourage the mother to stop feeding for a few minutes and comfort the infant.

A 12-month-old is admitted with a respiratory infection and possible pneumonia. He is placed in a mist tent with oxygen. Which nursing intervention has the greatest priority?`

C. Have a bulb syringe readily available to remove secretions

The healthcare provider prescribes methylergonovine maleate for a postpartum client with uterine atony. What findings should indicate to the nurse to withhold the next dose of the medication?`

C. Hypertension.

The nurse is assessing a 2-year-old. Which behavior indicates that the child's language development is within normal limits.`

C. Is capable of making a three word sentence

The nurse is counseling a woman who wants to become pregnant. The woman tells the nurse that she has a 36-day menstrual cycle and the first day of her last menstrual period was January 8. The nurse correctly calculates that the woman's next fertile period is`

C. January 30-31

When caring for a child with congenital heart disease and polycythemia, which nursing intervention has the highest priority?`

C. Maintaining adequate hydration

A client who delivered an infant an hour ago tells the nurse the she feels wet underneath her buttock. The nurse notes that the perineal pad is saturated and the client is lying in a 6-inch diameter pool of blood. Which action should the nurse implement first?`

C. Palpate the firmness of the fundus

The nurse is reviewing the health care provider's (HCP's) prescriptions for a client admitted for premature rupture of the membranes. Gestational age of the fetus is determined to be 37 weeks. Which prescription should the nurse question?`

C. Perform a vaginal examination every shift.

Six hours after an oxytocin (Pitocin) induction was begun and 2 hours after spontaneous rupture of the membranes, the nurse notes several sudden decreases in the fetal heart rate with quick return to baseline, with and without contractions. Based on this fetal heart rate pattern, which intervention is best for the nurse to implement?`

C. Place the client in a slight Trendelenburg position.

In developing a teaching plan for a 5-year-old child with diabetes, which component of diabetic management should the nurse plan for the child to manage first.`

C. Process of glucose testing

The nurse is developing a plan of care for a preterm newborn infant. The nurse develops measures to provide skin care, knowing that the preterm newborn infant's skin appears in what way?`

C. Reddened, translucent, and gelatinous, with decreased amounts of subcutaneous fat

A 26-year-old, gravida 2, para 1 client is admitted to the hospital at 28-weeks gestation in preterm labor. She is given 3 doses of terbutaline sulfate (Brethine) 0.25 mg subcutaneously to stop her labor contractions. The nurse plans to monitor for which primary side effect of terbutaline sulfate? `

C. Tachycardia and a feeling of nervousness.

A client at 30 weeks of gestation is on bed rest at home because of increased blood pressure. The home health nurse has taught her how to take her own blood pressure and gave her parameters to judge a significant increase in blood pressure. When the client calls the clinic complaining of indigestion, which instruction should the nurse provide?`

C. Take your blood pressure now, and if it is seriously elevated, go to the hospital.

A new mother who has just had her first baby says to the nurse, "I saw the baby in the recovery room. She sure has a funny looking head." Which response by the nurse is best? `

C. That is normal; the head will return to a round shape within 7 to 10 days.

The nurse discontinues a continuous IV heparin infusion for a male client on strict bed rest, and is now preparing to administer the client's first dose of in enoxaparin (Lovenox). Prior to giving this subcutaneous injection, which assessment finding requires additional intervention by the nurse?`

C. The client states that his right calf is aching, and wants pain medication.

The nurse performs a prescribed neurological check at the beginning of the shift on a client who was admitted to the hospital with a subarachnoid brain attack (stroke). The client's Glasgow Coma Scale (GCS) score is 9. What information is most important for the nurse to determine?`

C. The client's previous GCS score.

Prophylactic antibiotics are prescribed for a child who has mitral valve damage. The nurse should advise the parents to give the antibiotics prior to which occurrence?`

C. Urinary catheterization

A 14-year-old female client tells the nurse that she is concerned about the acne she has recently developed. Which recommendation should the nurse provide?`

C. Wash the hair and skin frequently with soap and hot water

A child falls on the playground and is brought to the school nurse with a small laceration on the forearm. Which action should the nurse implement first?`

C. Wash the wound gently with mild soap and water

A 4 week old premature infant has been receiving epoetin alfa for the last 3 weeks. WHich assessment finding indicates to the nurse that the drug is effective. `

C. changes in apica HR from the 180 to the 140

Heartburn and flatulence, common in the second trimester, are most likely the result of which of the following?`

C. decrease gastric acidity

The hormone responsible for the development of the ovum during the menstrual cycle is?`

C. follicle stimulating hormone

A client who delivered an infant an hour ago tells the nurse that she feels wet underneath her buttock. The nurse notes that both perineal pads are completely saturated and the client is lying in a 6inch diameter pool of blood.`

C. palpate the firmness of the fundus

The nurse must prevent a 2-year old with severe eczema on the face, neck and scalp from scratching the affected areas. Which nursing intervention is most effective in preventing further excoriation due to the pruritis? `

C. place elbow restraints on the child's arms

When talking with a pregnant client who is experiencing aching swollen, leg veins, the nurse would explain that this is most probably the result of which of the following? `

C. pressure on blood vessels from the enlarging uterus

Cervical softening and uterine souffle are classified as which of the following?`

C. probable signs

The nurse should encourage the laboring patient to begin pushing when`

C. the cervix is completely dilated

A client at 32-weeks gestation is hospitalized with severe pregnancy-induced hypertension (PIH), and magnesium sulfate is prescribed to control the symptoms. Which assessment finding indicates the therapeutic drug level has been achieved? `

C.A decrease in respiratory rate from 24 to 16.

A newborn infant is brought to the nursery from the birthing suite. The nurse notices that the infant is breathing satisfactorily but appears dusky. What action should the nurse take first? `

C.Check the infant's oxygen saturation rate.

A mother who is breastfeeding her baby receives instructions from the nurse. Which instruction is most effective to prevent nipple soreness? `

C.Correctly place the infant on the breast.

The total bilirubin level of a 36-hour, breastfeeding newborn is 14 mg/dl. Based on this finding, which intervention should the nurse implement? `

C.Encourage the mother to breastfeed frequently.

The nurse is teaching breastfeeding to prospective parents in a childbirth education class. Which instruction should the nurse include as content in the class? `

C.Feed your baby every 2 to 3 hours or on demand, whichever comes first.

At 14-weeks gestation, a client arrives at the Emergency Center complaining of a dull pain in the right lower quadrant of her abdomen. The nurse obtains a blood sample and initiates an IV. Thirty minutes after admission, the client reports feeling a sharp abdominal pain and a shoulder pain. Assessment findings include diaphoresis, a heart rate of 120 beats/minute, and a blood pressure of 86/48. Which action should the nurse implement next?`

C.Increase the rate of IV fluids.

A client at 30-weeks gestation, complaining of pressure over the pubic area, is admitted for observation. She is contracting irregularly and demonstrates underlying uterine irritability. Vaginal examination reveals that her cervix is closed, thick, and high. Based on these data, which intervention should the nurse implement first? `

C.Obtain a specimen for urine analysis.

Twenty minutes after a continuous epidural anesthetic is administered, a laboring client's blood pressure drops from 120/80 to 90/60. What action should the nurse take? `

C.Place the woman in a lateral position.

A couple, concerned because the woman has not been able to conceive, is referred to a healthcare provider for a fertility workup and a hysterosalpingography is scheduled. Which postprocedure complaint indicates that the fallopian tubes are patent? `

C.Shoulder pain.

The nurse is assessing the umbilical cord of a newborn. Which finding constitutes a normal finding? `

C.Three vessels: two arteries and one vein.

When preparing a class on newborn care for expectant parents, what content should the nurse teach concerning the newborn infant born at term gestation? `

C.Vernix is a white, cheesy substance, predominantly located in the skin folds.

The nurse should encourage the laboring client to begin pushing when `

C.the cervix is completely dilated.

A mother is concerned because she has counted only 8 kicks from her baby in two, 2-hour intervals; what intervention is necessary?

CALL THE HCP

One hour after giving birth to an 8-pound infant, a client's lochia rubra has increased from small to large and her fundus is boggy despite massage. The client's pulse is 84 beats/minute and blood pressure is 156/96. The healthcare provider prescribes Methergine 0.2 mg IM X 1. What action should the nurse take immediately?

Call the heathcare provider to question the prescription

Twenty-four hours after admission to the newborn nursery, a full-term male infant develops localized swelling on the right side of his head. In a newborn, what is the most likely cause of this accumulation of blood between the periosteum and skull that does not cross the suture line?

Cephalhematoma, which is caused by forceps trauma

A 4-week old premature infant has been receiving epoetin alfa (Epogen) for the last tree weeks. Which assessment finding indicates to the nurs that the drug is effective?

Change in indirect bilirubin from 12mg/dl to 8mg/dl

A newborn infant iss brought to the nursery from the birthing suite. The nurse notices that the infant is breathing satisfactory but appears dusky. What action should the nurse take first?

Check the infant's oxygen saturation rate

A full term infant is admitted to the newborn nursery. After careful assessment, the nurse suspects that the infant may have an esophageal atresia. Which symtoms are this newborn likely to exhibit?

Chocking, coughing, and cyanosis

A client at 28-weeks gestation calls the antepartal clinic and states that she is experiencing a small amount of vaginal bleeding which she describes as bright red. She further states that she is not experiencing any uterine contractions or abdominal pain. What instructions should the nurse provide?

Come into the clinic today for an ultrasound

A client at 28 weeks of gestation calls the antepartal clinic and states that she has just experienced a small amount of vaginal bleeding, which she describes as bright red. The bleeding has subsided. She further states that she is not experiencing any uterine contractions or abdominal pain. What instruction should the nurse provide?

Come to the clinic today for an ultrasound.

A multigravida client arrives at the labor and delivery unit and tell the nurse that her "bag of water" has broken. The nurse identifies the presence of meconium fluid on the perineum and determines the fetal heart rate is between 140-150 beats/minute. What action should the nurse implement next?

Complete a sterile vaginal exam

A multigravida client arrives at the labor and delivery unit and tell the nurse that her "bag of water" has broken. The nurse identifies the presence of meconium fluid on the perineum and determines the fetal heart rate is between 140 to 150 bpm. What action should the nurse implement next?

Complete a sterile vaginal exam.

A pregnant client with Mitral stenosis class III is prescribed complete bedrest. The client asks the nurse, "Why must I stay in bed all the time?" Which response is best for the nurse to provide this client?

Complete bedrest decreases oxygen needs and demands on the heart muscle tissue

The nurse is evaluating a full-term multigravida who was induced 3 hours ago. The nurse determines that the client is dilated 7 cm and is 100% effaced at 0 station, with intact membranes. The monitor indicates that the FHR decelerates at the onset of several contractions and returns to baseline before each contraction ends. Which action should the nurse take?

Continue to monitor labor progress.

Which findings are of most concern to the nurse when caring for a woman in the first trimester of pregnancy? (Select all that apply.)

Cramping with bright red spotting Lack of tenderness of the breast Increased right-side flank pain

A child is scheduled for a tonsillectomy in a day surgical unit. On the day after surgery, the mother calls the surgical unit and expresses concern because the child has a bad mouth odor. Which response is most appropriate?`

D. "Bad mouth odor is normal and may be relieved by drinking more liquids."

31) During the prenatal visit, the client states that she has been experiencing heartburn frequently. The LPN provides instruction on the cause and prevention of heartburn. When she ask to verbalize understanding of the information, which of the following statements by the client indicates further instruction may be necessary?`

D. "I'll eat enough to ensure that I am full at every meal."

The nurse provided discharge instructions to the parents of a 2-year-old child who had an orchiopexy to correct cryptorchidism. Which statement by the parents indicate that further teaching is necessary?`

D. "I'll let him decide when to return to his play activities."

A new mother is afraid to touch her baby's head for fear of hurting the "large soft spot." Which explanation should the nurse give to this anxious client?`

D. "There's a strong, tough membrane there to protect the baby so you need not be afraid to wash or comb their hair"

A nulliparous woman asks the nurse when she will begin to feel fetal movements. The nurse responds by telling the woman that the first recognition of fetal movement will occur at approximately how many weeks of gestation?`

D. 18 weeks

A pregnant woman comes to the prenatal clinic for an initial visit. In reviewing her childbearing hx, the client indicates that she has delivered premature twins, one full term baby and has had no abortions. Which GTPAL should the nurse document in this client's record?`

D. 3-1-1-0-3

Pediazole is a suspension medication that contains 200 mg erythromycin and 600 mg sulfisoxazole per 5 mL. The physician orders Pediazole 4 mL PO every 12 hours. How many mg of sulfisoxazole is this client receiving in a 24-hour period?`

D. 960 mg

When assessing a client who is at 12 week gestation, the nurse recommends that she and her husband consider attending childbirth preparation classes. When is the best time for the couple to attend these classes?`

D. At 30 weeks gestation.

A child is seen in the health care clinic for complaints of fever. On data collection, the nurse notes that the child is pale, tachycardic, and has petechiae. Aplastic anemia is suspected. The nurse understands that which diagnostic test will confirm the diagnosis of aplastic anemia?`

D. Bone marrow Biopsy

A mother calls the clinic because her 6-year-old son, who has been taking prescribed antibiotics for 7 of the previous 10 days, continues to have a cough that she reports is worsening. Further questioning by the nurse reveals that the cough is nonproductive. What advice should the nurse provide to this mother?`

D. Bring the child to the clinic today for an examination related to the cough.

One hour after giving birth to an 8-pound infant, a client's lochia rubra has increased from small to large and her fundus is boggy despite massage. HR is 84 bpm, BP 156/96. The M.D. prescribe Methergine 0.2 mg IM x 1. Which action should the nurse take immediately?`

D. Call the HP to question the prescription

What action should the nurse implement to decrease the client's risk for hemorrhage after c-section. `

D. Check the firmness of the uterus every 15mins.

A 2-year-old with gastroesophageal reflux disease has developed a fear of eating. What instruction should the RN include in the parent's teaching plan?`

D. Consistently follow a set meal-time routine

In making the initial assessment of a 2-hour-old infant, which finding should lead the nurse to suspect a congenital heart defect?`

D. Diminished femoral pulses

An infant is born to a mother with hepatitis B. Which prophylactic measure would be indicated for the infant?`

D. Hepatitis B immune globulin (HBIG) and hepatitis B vaccine given within 12 hours after birth

A 41-week multigravida is receiving oxytocin (Pitocin) to augment labor. Contractions are firm and occurring every 5 minutes, with a 30- to 40-second duration. The fetal heart rate increases with each contraction and returns to baseline after the contraction. Which action should the nurse implement?`

D. Increase the rate of the oxytocin (Pitocin) infusion.

An expectant father tells the nurse he fears that his wife "is losing her mind." He states she is constantly rubbing her abdomen and talking to the baby, and that she actually reprimands the baby when it moves too much. What recommendation should the nurse make to this expectant father?`

D. Let him know that these are normal maternal/fetal bonding behaviors which occur once the mother feels fetal movement

What preoperative nursing intervention should be included in the plan of care for an infant w/ pyloric stenosis?`

D. Observe for projectile vomiting

One hour following a normal vaginal delivery, a newborn infant boy's axillary temperature is 96° F, his lower lip is shaking, and when the nurse assesses for a Moro reflex, the boy's hands shake. Which intervention should the nurse implement first?`

D. Obtain a serum glucose level.

The nurse assesses a client admitted to the labor and delivery unit and obtains the following data: dark red vaginal bleeding, uterus slightly tense between contractions, BP 110/68, FHR 110 beats/minute, cervix 1 cm dilated and uneffaced. Based on these assessment findings, what intervention should the nurse implement?`

D. Perform Leopold's maneuvers

The nurse assists in the vaginal delivery of a newborn infant. After the delivery, the nurse observes the umbilical cord lengthen and a spurt of blood from the vagina. The nurse documents these observations as signs of which condition?`

D. Placental separation

An off-duty nurse finds a woman in a supermarket parking lot delivering an infant while her husband is screaming for someone to help his wife. Which intervention has the highest priority?`

D. Put the newborn to breast

The nurse caring for a client with a diagnosis of subinvolution should understand that which is a primary cause of this diagnosis?`

D. Retained placental fragments from delivery

What bowel habits are seen in children with Hirschsprung's disease?`

D. Ribbon-like and brown

After each feeding, a 3-day-old newborn is spitting up large amounts of Enfamil® Newborn Formula, a nonfat cow's milk formula. The pediatric healthcare provider changes the neonate's formula to Similac® Soy Isomil® Formula, a soy protein isolate based infant formula. What information should the nurse provide to the mother about the newly prescribed formula? `

D. Similac® Soy Isomil® Formula is a soy-based formula that contains sucrose.

A child with cystic fibrosis is having stools that float and are foul smelling. Which descriptive term should the nurse use to document the finding?`

D. Steatorrhea

A woman who had a miscarriage 6 months ago becomes pregnant. Which instruction is most important for the nurse to provide this client? `

D. Take prescribed multivitamin and mineral supplements.

A father of a 5-year-old boy calls the nurse to report that his son, who has had an upper respiratory infection, is complaining of a headache, and his temperature has increased to 103° F, taken rectally. Which intervention has the highest priority?`

D. Tell the parent to take the child to the emergency department.

In developing a teaching plan for expectant parents, the nurse decides to include information about when the parents can expect the infant's fontanels to close. Which statement is accurate regarding the timing of closure of an infant's fontanels that should be included in this teaching plan?`

D. The anterior fontanel closes at 12 to 18 months and the posterior fontanel by the end of the second month.

The LPN has initiated the administration of vancomycin via IV piggyback . In which of the following situations should the nurse recognize that the client may be experiencing a fatal reaction to this medication?`

D. The nurse notices the client's neck and chest is bright red

A couple has been trying to conceive for nine months without success. Which information obtained from the clients is most likely to have an impact on the couple's ability to conceive a child?`

D. They use lubricants with each sexual encounter to decrease friction

A mother discovers a bug has flown into her child's ear and can hear the buzzing; what intervention is priority?`

D. Use a flashlight to coax the insect out of the ear.

The nurse is monitoring a client in labor. The nurse suspects umbilical cord compression if which is noted on the external monitor tracing during a contraction?`

D. Variable decelerations

A client is pregnant with her third child. Medical history of the client indicates a previous precipitate labor and birth. Which of the following interventions would NOT be expected during labor of the present pregnancy?`

D. amnioinfusion will be performed

In developing a teaching plan for expectant parents the nurse plans to include formation about when the parents can expect the infants fontanels to close. The nurse bases the explanation on knowledge that for the normal newborn, the `

D. anterior fontanel closes at 12 to 18 months and the posterior by the end of the second month

In developing a teaching plan for expectant parents, the nurse plans to include information about when the parents can expect the infant's fontanels to close. The nurse bases the explanation on knowledge that for the normal newborn, the `

D. anterior fontanel closes at 12 to 18 months and the posterior by the end of the second month.

A 40 week gestation primigravida client is being induced with an ocytocin secondary infusion and complains of pain in her lower back. Which intervention should the nurse implement?`

D. apply firm pressure to sacral area

Which assessment finding should the nursery nurse report to the pediatric healthcare provider?`

D. central cyanosis when crying

During which of the following phase of the menstrual cycle is it ideal for implantation of a fertilized egg to occur? `

D. secretory phase

When assessing a client who is at 12-weeks gestation, the nurse recommends that she and her husband consider attending childbirth preparation classes. When is the best time for the couple to attend these classes? `

D.At 30-weeks gestation.

An expectant father tells the nurse he fears that his wife "is losing her mind." He states she is constantly rubbing her abdomen and talking to the baby, and that she actually reprimands the baby when it moves too much. What recommendation should the nurse make to this expectant father? `

D.Let him know that these behaviors are part of normal maternal/fetal bonding which occur once the mother feels fetal movement.

A client who gave birth to a healthy 8 pound infant 3 hours ago is admitted to the postpartum unit. Which nursing plan is best in assisting this mother to bond with her newborn infant? `

D.Meet the mother's physical needs and demonstrate warmth toward the infant.

Just after delivery, a new mother tells the nurse, "I was unsuccessful breastfeeding my first child, but I would like to try with this baby." Which intervention is best for the nurse to implement first? `

D.Provide assistance to the mother to begin breastfeeding as soon as possible after delivery.

An off-duty nurse finds a woman in a supermarket parking lot delivering an infant while her husband is screaming for someone to help his wife. Which intervention has the highest priority? `

D.Put the newborn to breast

A new mother is afraid to touch her baby's head for fear of hurting the "large soft spot." Which explanation should the nurse give to this anxious client? `

D.There's a strong, tough membrane there to protect the baby so you need not be afraid to wash or comb his/her hair.

A 35-year-old primigravida client with severe preeclampsia is receiving magnesium sulfate via continuous IV infusion. Which assessment data indicates to the nurse that the client is experiencing magnesium sulfate toxicity? `

D.Urine output 90 ml/4 hours.

A HCP informs the charge nurse of a labor and delivery unit that a client is coming to the unit with suspected abruption placentae. What findings should the nurse expect the client to demonstrate?

Dark, red vaginal bleeding. increased uterine irritability. A rigid abdomen

A client with no prenatal care arrives at the labor unit scraming, "The baby is coming!" The nurse preforms a vaginal examination that reveals the cervix is 3 centimeters dialated and 75% effaced. What additional information is most important for the nurse to obtain?

Date of last menstrual period.

A woman with type 2 diabetes mellitus becomes pregnant, and her oral hypoglycemic agents are discontinued. Which intervention is most important for the nurse to implement?

Describe diet changes that can improve the management of her diabetes

A client at 32-weeks gestation comes to the prenatal clinic with complaints of pedal edema, dyspnea, fatigue and a moist cough. Which question is most important for the nurse to ask this client?

Do you have a history of rheumatic fever?

A 24-hour old newborn has a pink popular rash with vesicles superimposed on the thorax, back and abdomen. What action should the nurse implement

Document the finding in the infants record.

A client who has an autosomal dominant inherited disorder is exploring family planning options and the risk of transmission of the disorder to an infant. The nurse's response should be based on what information?

Each pregnancy carries a 50% chance of inheriting the disorder.

The nurse is caring for a woman with a previously diagnosed heart disease who is in the second stage of labor. Which assessment findings are of greatest concern?

Edema, basilar rales, and irregular pulse

The nurse is planning preconception care for a female client. Which information should the nurse provide the client?

Encourage healthy lifestyles for families desiring pregnancy

The total bilirubin level of a 36-hour, breastfeeding newborn is 14mg/dl. Based on this finding, which intervention should the nurse implement?

Encourage the mother to breast feed frequently

The Total bilirubin of a 36-hour, breastfeeding newborn is 14mg/dL. Based on this finding, Which intervention should the nurse implement?

Encourage the mother to breastfeed frequently.

A new mother is having trouble breastfeeding her newborn. The child is making frantic rooting motions and will not grasp the nipple. Which intervention should the nurse implement?

Encourage the mother to stop feeding for a few minutes and comfort the infant.

A mother who is breastfeeding her baby receives instructions from the nurse. Which instruction is most effective in preventing nipple soreness?

Ensure that the baby is positioned correctly for latching on.

A client at 32-weeks gestation is diagnosed with preeclampsia. Which assessment finding is most indicative of an impending convulsion>

Epigastric pain

A client who delivered a healthy infant 5 days ago calls the clinic nurse and reports that her lochia is getting lighter in color. Which action should the nurse take?

Explain this is a normal finding.

A client is active labor complains of cramps in her leg. What intervention should the nurse implement?

Extend the leg and dorsiflex the foot

The nurse is teaching breastfeeding to prospective parents in a childbirth education calss.. Which instruction should the nurse include as content in the class?

Feed your baby every 2 to 3 hours or on demand, whichever comes first.

The nurse is preparing a laboring client for an amniotomy. Immediately after the procedure is completed, it is most important for the nurse to obtain which information?

Fetal heart rate (FHR)

The healthcare provider prescribes terbutaline (Brethine) for a clint in preterm labor. Before initiating this prescription, it is most important for the nurse to assess the client for which condition.

Gestational diabetes

The nurse is teaching care of the newborn to a childbirth preparation class and describes the need for administering antibiotic ointment into the eyes of the newborn. An expectant father asks, "What type of disease causes infections in babies that can be prevented by using this ointment?" Which response by the nurse is accurate?

Gonorrhea

The nurse is teaching care of the newborn to a group of prospective parents and describes the need for administering antibiotic ointment into the eyes of the newborn which infectious organism will this treatment prevent from harming the infant?

Gonorrhea

A 25-year-old client has a positive pregnancy test. One year ago she had a spontaneous abortion at 3 months of gestation. Which is the correct description of this client that should be documented in the medical record?

Gravida 2, para 0

The nurse attempts to help and unmarried teenager deal with her feelings following a spontaneous abortion at 8-weeks gestation. What type of emotional response should the nurse anticipate?

Grief related to her perceptions about loss of this child.

A mother who is HIV-positive delivers a full-term newborn and asks the nurse if her baby will become HIV-infected. Which explanation should the nurse provide?

HIV infection is determined at 18 months of age, when maternal HIV antibodies are no longer present.

What does a brace do for scoliosis?

Halts the progression of most curvatures. DOES NOT FIX CURVATURE.

During the transition phase of labor, a client complains of tingling and numbness in her fingers and tells the nurse that she feels like she is going to pass out. What action should the nurse take?

Have her cup both hands over her nose and mouth while breathing.

The nurse instructs a laboring client to use accelerated blow breathing. The client begins to complain of tingling fingers and dizziness. Which action should the nurse take?

Have the client breathe into her cupped hands.

Which action should the nurse implement when preparing to measure the fundal height of a pregnant client?

Have the client empty her bladder.

Which maternal behavior is the nurse most likely to see when a new mother receives her infant for the first time?

Her arms and hands receive the infant and she then traces the infant's profile with her fingertips.

A primigracida at 40-weeks gestation is receiving oxytocin to augment labor. Which adverse effect should the nurse monitor for during the infusion of Pitocin?

Hyperstimulation

A primigravida, when returning for the results of her multiple marker screening (triple screen), asks the nurse how problems with her baby can be detected by the test. What information will the nurse give to the client to describe best how the test is interpreted?

If MSAFP and estriol levels are low and the hCG level is high, results are positive for a possible chromosomal defect.

A client who delivered by cesarean section 24 hours ago is using a patient-controlled analgesia (PCA) pump for pain control. Her oral intake has been ice chips only since surgery. She is now complaining of nausea and bloating and states that because she has had nothing to eat, she is too weak to breastfeed her infant. Which nursing diagnosis has the highest priority?

Impaired bowel motility related to pain medication and immobility

A 41-week multigravida is receiving oxytocin (Pitocin) to augment labor. Contractions are firm and occurring every 5 minutes, with a 30- to 40-second duration. The fetal heart rate increases with each contraction and returns to baseline after the contraction. Which action should the nurse implement?

Increase the rate of the oxytocin (Pitocin) infusion.

Which findings are most critical for the nurse to report to the primary health care provider when caring for the client during the last trimester of her pregnancy? (Select all that apply.)

Increased heartburn that is not relieved with doses of antacids Chronic headache that has been lingering for a week behind the client's eyes

A full term infant is transferred to the nursery from the labor and delivery. Which information is most important for the nurse to receive when planning immediate care for the newborn?

Infant's condition at birth and treatment received

Immediately after birth a newborn infant is suctioned, dried and placed under a radiant warmer. The infant has spontaneous respirations and the nurse assesses an apical heart rate of 80 beats/minute and respirations of 20 breaths/minute. What action should the nurse preform next?

Initiate positive pressure ventilation

A client who is attending antepartum classes asks the nurse why her healthcare provider has prescribed iron tablets. The nurse's response based on what knowledge?

It is difficult to consume 18mg of additional iron by diet alone

A client who is in the second trimester of pregnancy tells the nurse that she wants to use herbal therapy. Which response is best for the nurse to provide?

It is important that you want to take part in your care

The nurse is counseling a client who wants to become pregnant. She tells the nurse that she has a 36-day menstrual cycle and the first day of her last menstrual period was January 8. When will the client's next fertile period occur?

January 29 to 30

The nurse is counseling a woman who wants to become pregnant. The woman tells the nurse that she has a 36-day menstrual cycle and the first day of her last menstrual period was January 8. The nurse correctly calculates that the woman's next fertile period will be

January 30-31

This is a dark streak down the midline of the abdomen that may appear as the uterus is enlarging. The LPN correctly describes this to the pregnant woman as?

LINEA NIGRA

During a prenatal visit, the nurse discusses the effects of smoking on the fetus with a client. Which statement is most characteristic of an infant whose mother smoked during pregnancy compared with the infant of a nonsmoking mother?

Lower initial weight documented at birth

Which nursing intervention would be most helpful in relieving postpartum uterine contractions or "afterpains?"

Lying prone with a pillow on the abdomen

A female client with insulin-dependent diabetes arrives at the clinic seeking a plan to get pregnant in approximately 6 months. She tells the nurse that she wants to have an uncomplicated pregnancy and a healthy baby. What information should the nurse share with the client?

Maintain blood sugar levels in a constant range within normal limits during pregnancy

A 30-year-old gravida 2, para 1 client is admitted to the hospital at 26-weeks gestation in preterm labor. She is started on an IV solution of tervutaline (Berthine). Which assessment is the highest priority for the nurse to monitor during the administration of this drug?

Maternal and fetal heart rates.

A pregnant client tells the nurse that the first day of her last menstrual period was August 2, 2006. Based on Ngele's rule, what is the estimated date of delivery?

May 9, 2006

A pregnant client tells the nurse that the first day of her last menstrual period was Aug. 2, 2006. Based on Ngele's rule, what is the estimated date of delivery?

May 9, 2007

The nurse assesses a client admitted to the labor and delivery unit and obtains the following data: dark red vaginal bleeding, uterus slightly tese between contractions, BP 110/68 FHR 110 beats/min, cervix 1 cm dialated and uneffaced. Based on these assessment findings, what intervention should the nurse implement?

Monitor bleed from IV sites

When explaining "postpartum blues" to a client who is 1 day postpartum, which symtoms should the nurse include in the teaching plan?(Select all that apply)

Mood swings Tearfulness

A 38-week primigravida who works as a secretary and sits at a computer 8 hours each day tells the nurse that her feet have begun to swell. Which instruction will aid in the prevention of pooling of blood in the lower extremities?

Move about every hour

During a prenatal visit, the nurse discusses with a client the effects of smoking on the fetus. When compared with nonsmokers, mothers who smoke during pregnancy tend to produce infants who have `

Move about every hour.

The nurse observes that an antepartum client who is on bed rest for preterm labor is eating ice rather than the food on her breakfast tray. The client states that she has a craving for ice and then feels too full to eat anything else. Which is the best response by the nurse?

Notify the health care provider

On admission to the prenatal clinic, a 23 year old woman tells the nurse that her last menstrual period began on Feb 15, and that her previous periods were regular. Her pregnancy test is positive. The client's expected date of delivery (EDD) would be

November 22

On admission to the prenatal clinic, a client tells the nurse that her last menstrual period began on February 15 and that previously her periods were regular (28-day cycle). Her pregnancy test is positive. What is this client's expected date of birth (EDB)?

November 22

The nurse identifies crepitus when examining the chest of a newborn who was delivered vaginally which further assessment should the nurse perform?

Observe for an asymmetrical Moro reflex

A newborn infant, diagnosed with developmental dysplasia of the hip (DDH), is being prepared for discharge. Which nursing intervention should be included in this infant's discharge teaching plan?

Observe the parents applying a Pavlik harness.

One hour following a normal vaginal delivery, a newborn infant boy's axillary temperature is 96° F, his lower lip is shaking, and when the nurse assesses for a Moro reflex, the boy's hands shake. Which intervention should the nurse implement first?

Obtain a serum glucose level.

A client at 30-weeks gestation, complaining of pressure over the pubic area, is admitted for observation. She is contracting irregularly and demonstrates underlying uterine irritability. Vaginal examination reveals that her cervix is close, thick, and high. Based on these data, which intervention should the nurse implement first?

Obtain a specimen for urine

A client is admitted with the diagnosis of total placenta previa. Which finding is most important for the nurse to report to the healthcare provider immediately?

Onset of uterine contractions

A client is admitted with the diagnosis of total placenta previa. Which finding is most important for the nurse to report to the HCP?

Onset of uterine contractions.

A client is admitted with the diagnosis of total placenta previa. Which finding is most important for the nurse to report to the healthcare provider immediately. `

Onset of uterine contractions.

The client comes to the hospital assuming she is in labor. Which assessment findings by the nurse would indicate that the client is in true labor? (Select all that apply.)

Pain in the lower back that radiates to abdomen Progressive cervical dilation and effacement Regular and rhythmic painful contractions

A client in active labor is admitted with preeclamsia. Which assessment finding is most significant in planning this client's care?

Patellar reflex 4+

A mother expresses fear about changing the infant's diaper after circumcision. What information should the nurse include in the teaching plan?

Place petroleum ointment around the glans with each diaper change and cleansing

Twenty minutes after a continuous epidural anesthetic is administered, a laboring client's blood pressure drops from 120/80 to 90/60 mm Hg. Which action should the nurse take immediately?

Place the client in a lateral position.

Six hours after an oxytocin (Pitocin) induction was begun and 2 hours after spontaneous rupture of the membranes, the nurse notes several sudden decreases in the fetal heart rate with quick return to baseline, with and without contractions. Based on this fetal heart rate pattern, which intervention is best for the nurse to implement?

Place the client in a slight Trendelenburg position.

Twenty minutes after a continuous epidural anesthetic is administered, a laboring client's blood pressure drops from 120/80 to 90/60. What action will the nurse take

Place the woman in lateral position

The nurse observes a new mother is rooming in and caring for her newborn infant. Which observation indicated the need for further teaching?

Places the infant prone in the bassinet

Ativan 0.5 mg IM every 1 hour as needed is prescribed for a client experiencing delirium tremens. The medication vial reads 2mg/mL of solution. How many mL should the LPN draw into the syringe for single dose administration?

Possible correct answers: 0.25 mL0.25mL0.25ml0.25 ml Explanation 2mg/mL= 0.5mg/xmL 2x=0.5 x=0.5/2 x=0.25 mL

The nurse calls a client who is 4 days postpartum to follow up about her transition with her newborn son at home. The woman tells the nurse, "I don't know what is wrong. I love my son, but I feel so let down. I seem to cry for no reason!" Which adjustment phase should the nurse determine the client is experiencing?

Postpartum blues

Just after delivery, a new mother tells the nurse, "I was unsuccessful breastfeeding my first child, but I would like to try with this baby." Which intervention should the nurse implement first?

Provide assistance to the mother to begin breastfeeding as soon as possible after delivery

Just after delivery, a new mother tells the nurse, "I was unsuccessful breastfeeding my first child, but I would like to try with this baby." Which intervention is best for the nurse to implement First?

Provide assistance to the mother to begin breastfeeding as soon as possible after delivery.

An off-duty nurse finds a woman in a supermarket parking lot delivering an infant while her husband is screaming for someone to help his wife. Which intervention has the highest priority?

Put the newborn to the breast immediately

A client receiving an epidural anesthesia begins to experience nausea and becomes pate and clammy. What intervention should the nurse implement first?

Raise the foot of the bed

A 350-bed acute care hospital declares an internal disaster because the emergency generators malfunctioned during a city-wide power failure. The Unlicensed Assistive Personnel (UAP)s working on a general medical unit ask the charge nurse what they should do first. What instruction should the charge nurse provide to these UAPs?`

Rationale D. Tell all their assigned clients to stay in their rooms. Rationale

An expectant father tells the nurse he fears that his wife "is losing her mind." He states that she is constantly rubbing her abdomen and talking to the baby and that she actually reprimands the baby when it moves too much. Which recommendation should the nurse make to this expectant father?

Reassure him that normal maternal-fetal bonding is occurring

The nurse is providing discharge teaching for a client who is 24 hours post partum. The nurse explains to the client that her vaginal discharge will change from red to pink then to white. The client asks, "What if I start having red bleeding after it changes?" What should the nurse instruct the client to do?

Reduce activity level and notify the healthcare provider.

A 30-year-old multiparous woman who has a 3-year old boy and a newborn girl tells the nurse, "my son is so jealous of my daughter, I don't know how ill ever manage both children when I get home." How should the nurse respond?

Regression in behaviors in the older child is a typical reaction so he needs attention at this time

During labor, the nurse determines that a full-term client is demonstrating late decelerations. In which sequence should the nurse implement theses actions? (Place the first action on top and the last on the bottom) Provide oxygen via mask reposition client call HCP Increase IV Fluids

Reposition the client Increase IV Fluids provide oxygen via face mask Call HCP

The nurse should explain to a 30-year old gravid client that alpha fetoprotein testing is recommended for which purpose?

Screen for neural tube defects

After each feeding, a 3-day-old newborn is spitting up large amounts of Enfamil Newborn Formula, a nonfat cow's milk formula. The pediatric healthcare provider changes the neonate's formula to Similac Soy Isomil Formula, a soy protein isolate based infant formula. What information should the nurse provide to the mother about newly prescribed formula?

Similac Soy Isomil Formula is a soy based formula that contains sucrose.

Which statement made by the client indicates that the mother understands the limitations of breastfeeding her newborn?

"Breastfeeding my infant consistently every 3 to 4 hours stops ovulation and my period."

A client at 32-weeks gestation comes to the prenatal clinic with complaints of pedal edema, dyspnea, fatigue, and a moist cough. Which question is most important for the nurse to ask this client?

"Do you have a history of rheumatic fever?"

A nurse receives a shift change report for a newborn who is 12 hours post-vaginal delivery. In developing a plan of care, the nurse should give the highest priority to which finding?

Skin color that is slightly jaundiced

A 26-year-old gravida 2, para 1, client is admitted to the hospital at 28 weeks of gestation in preterm labor. She is given three doses of terbutaline sulfate (Brethine), 0.25 mg subcutaneously, to stop her labor contractions. What are the primary side effects of terbutaline sulfate?

Tachycardia and a feeling of nervousness

A woman who had a miscarriage 6 months ago becomes pregnant. Which instruction is most important for the nurse to provide this client?

Take prescribed multivitamin and mineral supplements

A client at 30 weeks of gestation is on bed rest at home because of increased blood pressure. The home health nurse has taught her how to take her own blood pressure and gave her parameters to judge a significant increase in blood pressure. When the client calls the clinic complaining of indigestion, which instruction should the nurse provide?

Take your blood pressure now, and if it is seriously elevated, go to the hospital.

A 23- year old client who is receiving Medicid benefits is pregnant with her first child. Based on knowledge of the statistics related to infant mortality, which plan should the nurse implement with this client?

Teach the client why keeping prenatal care appointments is important

In developing a teaching plan for expectant parents, the nurse decides to include information about when the parents can expect the infant's fontanels to close. Which statement is accurate regarding the timing of closure of an infant's fontanels that should be included in this teaching plan?

The anterior fontanel closes at 12 to 18 months and the posterior fontanel by the end of the second month.

Client teaching is an important part of the perinatal nurse's role. Which factor has the greatest influence on successful teaching of the pregnant client?

The client's investment in what is being taught

A female client with insulin-dependent diabetes arrives at the clinic seeking a plan to get pregnant in approximately 6 months. She tells the nurse that she want to have an uncomplicated pregnancy and a healthy baby. What information should the nurse share with the client?

"Maintain blood sugar levels in a constant rage within normal limits during pregnancy."

A new mother who has just had her first baby says to the nurse, "I saw the baby in the recovery room. She sure has a funny-looking head." Which response by the nurse is best?

"That is normal. The head will return to a round shape within 7 to 10 days."

A new mother asks the nurse, "How do I know that my daughter is getting enough breast milk?" Which explanation is appropriate?

"Your milk is sufficient if the baby is voiding pale, straw-colored urine six to ten times a day."

A 30-year-old multiparous woman who has a 3-year-old boy and an newborn girl tells the nurse, "My son is so jealous of my daughter, I don't know how I'll ever manage both children when I get home." How should the nurse respond? `

...

A 38-week primigravida who works as a secretary and sits at a computer 8 hours each day tells the nurse that her feet have begun to swell. Which instruction would be most effective in preventing pooling of blood in the lower extremities? `

...

During labor, the nurse determine that a full term client is demonstrating late decelerations. In which sequence should the nurse implement these nursing actions.

1. Reposition the client 2. Provide O2 via face mask 3. Increase IV fluid 4. Call the healthcare provider

What is the therapeutic level of theophylline?

10-20 mcg/dL

A client at 32-weeks gestation is diagnosed with preeclampsia. Which assessment finding is most indicative of impending convulsion?

3+ deep tendon reflexes and hyperclonus.

The nurse is using the Silverman-Anderson index to assess an infant with respiratory distress and determines that the infant is demonstrating marked nasal flaring, an audible expiratory grunt, and just visible intercostal and xiphoid retractions. Using this scale, which score should the nurse assign?

5

A 16-year-old adolescent with meningococcal meningitis is receiving a continuous IV infusion of penicillin G, which is prescribed as 20 million units in a total volume of 2 liters of normal saline every 24 hours. The pharmacy delivers 10 million units/ liter of normal saline. How many ml/hr should the nurse program the infusion pump? (Enter numeric value only. If rounding is required, round to the nearest whole number.)

83 ml/hour

The nurse is preparing to give an enema to a laboring client. Which client would require most caution when carrying out this procedure?

A 40-week primigravida who is at 6 cm cervical dilatation and the presenting part is not engaged.

A client 32 weeks gestation is hospitalized with severe pregnancy-induced hypertension (PIH), and magnesium sulfate is prescribed to control the symptoms. Which assessment finding would indicate the therapeutic drug level has been achieved?

A decrease in respiratory rate from 24 to 16

A woman who thinks she could be pregnant calls her neighbor, a nurse, to ask when she could use a home pregnancy test to diagnose pregnancy. Which response is appropriate?

A home pregnancy test can be used right after your first missed period

A newborn, whose mother is HIV positive, is scheduled for follow-up assessments. The nurse knows that the most likely presenting symptom for a pediatric client with AIDS is

A persistent cold

A primigravida client who is 5cm dilated, 90% effaced and 0 station is requesting an epidural for pain relief. Which assessment finding is most important for the nurse to report to the healthcare provider?

A platelet count of 67.000/mm3

A woman who thinks she could be pregnant calls her neighbor, who is a nurse, to ask when she should use a home pregnancy test. Which response is appropriate?`

A. "A home pregnancy test can be used right after your first missed period."

Which statement made by the client indicates that the mother understands the limitations of breastfeeding her newborn?`

A. "Breastfeeding my infant consistently every 3 to 4 hours stops ovulation and my period."

As part of the physical assessment of children, the nurse observes and palpates the fontanels. Which child's fontanel finding should be reported to the healthcare provider?`

A. A 6-month old with failure to thrive that has a closed anterior fontanel

A woman who thinks she could be pregnant calls her neighbor, a nurse, to ask when she could use a home pregnancy test to diagnose pregnancy. Which response is best? `

A. A home pregnancy test can be used right after your first missed period.

A nurse is caring for a client in the active stage of labor. The nurse notes that the fetal pattern shows a late deceleration on the monitor strip. Based on this finding the nurse should prepare for which appropriate nursing action?`

A. Administering oxygen via face mask

A nursing instructor asks a nursing student to list the functions of the amniotic fluid. The student responds correctly by stating that which of the following are functions of amniotic fluid? Select all that apply.`

A. Allows for fetal movement B. Is a measure of kidney function C. Surrounds, cushions, and protects the fetus D. Maintains the body temperature of the fetus

The nurse knows that there are psychological maternal changes that occurs during pregnancy in a primigravida patient. Select all the normal psychological maternal changes that happens throughout pregnancy. `

A. Ambivalence C. Emotional lability D. Body image changes E. Bonding or relationship with the fetus

The nurse is conducting an initial admission assessment of a 12-month-old child in celiac crisis. Which intervention is most important for the nurse to implement?`

A. Assess the child's mucous membranes and skin turgor.

Family centered nursing care for women and newborn focuses on which of the following?`

A. Assisting individuals and families achieve their optimal health

According to Diane, her LMP is November 15, 2002, using the Naegle's rule what is her EDC? `

A. August 23, 2003

The nurse is teaching a woman how to use her basal body temperature (BBT) pattern as a tool to assist her in conceiving a child. Which temperature pattern indicates the occurrence of ovulation, and therefore, the best time for intercourse to ensure conception? `

A. Between the time the temperature falls and rises.

A child is admitted to the hospital for confirmation of a diagnosis of acute lymphoblastic leukemia. During the initial nursing assessment, which symptoms will this child most likely exhibit?`

A. Bone pain, pallor

A breastfeeding postpartum client is diagnosed with mastitis, and antibiotic therapy is prescribed. Which instruction should the nurse provide to this client?`

A. Breastfeed the infant, ensuring that both breasts are completely emptied.

A full-term infant is admitted to the newborn nursery and, after careful assessment, the nurse suspects that the infant may have an esophageal atresia. Which symptoms is this newborn likely to have exhibited?`

A. Choking, coughing, and cyanosis

A client at 28-weeks gestation calls the antepartal clinic and states that she is experiencing a small amount of vaginal bleeding which she describes as bright red. She further states that she is not experiencing any uterine contractions or abdominal pain. Which instruction should the nurse provide?`

A. Come to the clinic today for an ultrasound

A 2-year-old child with Downs Syndrome is brought to the clinic for his regular physical exam. The nurse knows which problem is frequently associated with DS`

A. Congenital heart disease

Which findings are of most concern to the nurse when caring for a woman in the first trimester of pregnancy? (Select all that apply.)`

A. Cramping with bright red spotting C. Lack of tenderness of the breast E. Increased right-side flank pain

A woman with Type 2 diabetes mellitus becomes pregnant, and her oral hypoglycemic agents are discontinued. Which intervention is most important for the nurse to implement?`

A. Describe diet changes that can improve the management of her diabetes

A 36 weeks gestation pregnant woman is complaining of urinary urgency and frequency. The nurse explained that the enlarging fetus is pressing the bladder which causes frequent urination. This is normally occuring during the first and third trimesters of pregnancy. The nurse advices the patient to do the following measures to prevent urinary frequency. Select all the necessary measures that the nurse can provide to the patient. `

A. Drink 2 quarts of fluid during the day C. Performing Kegel exercises E. Limiting fluid intake during the evening

A primigravida patient who is 12 weeks pregnant visits a helath promotion program in the community pertaining to the pregnancy care. A group of nursing student is educating the public about measures to prevent discomfort of pregnancy. The primigravida patient asks one of the student about measures on how to prevent heartburn she is experiencing throughout the day. Select all the necessary measures to prevent the primigravia patient's complaint.`

A. Eating small, frequent meals and avoiding fatty and spicy food C. Drinking milk between milk E. Sitting upright for 30 minutes after a meal

A 6-year-old is admitted to the pediatric unit after falling off of a bicycle. Which intervention should the RN implement to help the child adjust to the unit.`

A. Explain hospital schedules, including mealtime

Which action should the nurse implement when preparing to measure the fundal height of a pregnant client?`

A. Have the client empty the bladder

A nurse is monitoring a client who is in the active phase of labor. The client has been experiencing contractions that are short, irregular, and weak. Which type of labor dystocia should the nurse document that the client is experiencing?`

A. Hypotonic

Which findings are most critical for the nurse to report to the primary health care provider when caring for the client during the last trimester of her pregnancy? (Select all that apply.)`

A. Increased heartburn that is not relieved with doses of antacids E. Chronic headache that has been lingering for a week behind the client's eyes

Immediately after birth a newborn infant is suctioned, dried and placed under a radiant warmer. The infant has spontaneous respirations and the nurse assesses an apical HR of 80 beats/minute and respirations of 20 breaths/min. What action should be performed next?`

A. Initiate positive pressure ventilation

When assessing a child with asthma, the nurse should expect intercostal retractions during`

A. Inspiration

Which nursing intervention is most helpful in relieving postpartum uterine contractions or "afterpains?" `

A. Lying prone with a pillow on the abdomen.

Which finding in a 19 yr old female client should trigger further assessment by the nurse?`

A. Menstruation has not occurred

A 3-week-old newborn is brought to a clinic for follow-up after a home birth. The child bottle feeds for 5 minutes only then falls asleep. A loud murmur characteristic of ventricular septal defect (VSD), and finds the newborn acyanotic with a RR of 64 bpm. What instruction should be provided to ensure the newborn gets adequate fluid intake? (Select all that apply).

A. Monitor the infant's weight and number of wet diapers per day B. Increase the infant's intake per feeding by 1-2 ounces per week D. Allow the infant to rest and refeed on demand or every 2 hours E. Use a softer nipple or increase the size of the nipple opening

When explaining "postpartum blues" to a client who is 1 day postpartum, which symptoms should the nurse include in the teaching plan? (Select all that apply)`

A. Mood swings C. Tearfulness

A 3-month-old develops oral thrush. Which pharmacological agent should the RN administer?`

A. Nystatin

Which menu selection by a child w/ celiac disease indicates to the nurse that the child understands necessary dietary considerations?`

A. Oven-baked potato chips and cola

When discussing discipline with the mother of a 4 year-old, the nurse should include which guideline?`

A. Parental control should be constant

A child diagnosed with scarlet fever is being cared for at home. The home health nurse performs an assessment on the child and checks for which clinical manifestations of this disease? Select all that apply.`

A. Pastia's sign C. White strawberry tongue D. Edematous and beefy-red pharynx E. Petechial red, pinpoint spots on the soft palate

A 3-month-old infant returns from surgery with elbow restraints and a Logan bow over a cleft lip suture line. Which intervention should the nurse implement to maintain suture line integrity during the initial postoperative period?`

A. Place the infant upright in an infant seat position.

The nurse is teaching a 12-year-old male adolescent and his family about taking injections of growth hormone for idiopathic hypopituitarism. Which adverse S/S are commonly associated with this therapy?`

A. Polyuria and polydipsia

The nurse reviews the laboratory results for a child with rheumatic fever and would expect to note which findings? Select all that apply.`

A. Presence of Aschoff's bodies C. Elevated antistreptolysin O titer E. Elevated erythrocyte sedimentation rate

A 2-year-old child with trisomy 21 (Down syndrome) is brought to the clinic for a routine evaluation. Which assessment finding suggests the presence of a common complication often experienced by those with Down syndrome?`

A. Presence of a systolic murmur

Which interventions should the nurse include when preparing a care plan for a child with hepatitis? Select all that apply`

A. Providing a low-fat, well-balanced diet. B. Teaching the child effective hand-washing techniques. E. Instructing the parents to avoid administering medications unless prescribed.

client receiving epidural anesthesia begins to experience nausea and becomes pale and clammy. What intervention should the nurse implement first? `

A. Raise the foot of the bed.

When evaluating the effectiveness of interventions to improve the nutritional status of an infant with gastroesophageal reflux disease, which intervention is most important for the nurse to implement?`

A. Record weight daily

A child comes to the school nurse complaining of itching. Further assessment reveals that the child has impetigo. What action should the nurse take?`

A. Send the child home with the parents to see the health care provider before returning to school.

Which assessment findings should the nurse expect when caring for a child with cystic fibrosis? (Select all that apply.)`

A. Steatorrhea C. Foul-smelling stools D. Delayed growth E. Pulmonary congestion

The nurse is reviewing the record of a pregnant client seen in the health care clinic for the first prenatal visit. Which data if noted on the client's record would alert the nurse that the client is at risk for developing gestational diabetes during this pregnancy?`

A. The client's last baby weighed 10 pounds at birth.

Client teaching is an important part of the maternity nurse's role. Which factor has the greatest influence on successful teaching of the gravid client? `

A. The client's readiness to learn.

Which of the following would cause a false-positive result on a pregnancy test?`

A. The test was performed less than 10 days after an abortion

A new mother who has just had her first baby says to the nurse, "I saw the baby in the recovery room. She sure has a funny looking head." Which response by the nurse is best?`

A. This is not an unusual shaped head. espec

The nurse is giving a liquid iron preparation to a 3-year-old, which technique should the RN implement to encourage active engagement?`

A. Use a colorful straw

To assess the effectiveness of an analgesic administered to a 4-yr old, what intervention is best for the nurse to implement?`

A. Use a happy-face/sad-face pain scale

Twenty-four hours after admission to the newborn nursery, a full-term male infant develops localized edema on the right side of his head. The nurse knows that, in the newborn, an accumulation of blood between the periosteum and skull which does not cross the suture line is a newborn variation known as`

A. a cephalhematoma, caused by forceps trauma and may last up to 8 weeks

A multigravida client arrives at the L&D unit and tells the nurse that her bag of water has broken. The nurse identifies the presence of meconium fluid on the perineum and determines the fetal HR is between 140 and 150 beats/min. What action should the nurse implement next?`

A. complete sterile vag exam

A healthcare provider informs the charge nurse of a labor and delivery unit that a client is coming to the unit suspected abruptio placentae. What findings should the charge nurse expect the client to demonstrate.`

A. dark,red vaginal bleeding D. increased uterine irritability F. Rigid abdomen

The healthcare provider prescribes terbutaline for a client in preterm labor. Before initating this prescription, it is most important for the nurse to assess the client for which of condition. `

A. gestational diabetes

A 42 week gestational client is receiving an intravenous infusion of oxytocin(Pitocin) to augment early labor. The nurse should discontinue the oxytocin infusion for with pattern of contractions?`

A. transition labor with contractions every 2 mins, lasting 90 seconds each.

The nurse is performing a gestational age assessment on a full-term newborn during the first hour of transition using the Ballard (Dubowitz) scale. Based on this assessment, the nurse determines that the neonate has a maturity rating of 40-weeks. What findings should the nurse identify to determine if the neonate is small for gestational age (SGA)? (Select all that apply.) `

A.Admission weight of 4 pounds, 15 ounces (2244 grams) B.Head to heel length of 17 inches (42.5 cm). C.Frontal occipital circumference of 12.5 inches (31.25 cm).

A vaginally delivered infant of an HIV positive mother is admitted to the newborn nursery. What intervention should the nurse perform first?

A.Bathe the infant with an antimicrobial soap. B.Measure the head and chest circumference. C. Obtain the infant's footprints. D. Administer vitamin K (AquaMEPHYTON).

A client at 28-weeks gestation calls the antepartal clinic and states that she is experiencing a small amount of vaginal bleeding which she describes as bright red. She further states that she is not experiencing any uterine contractions or abdominal pain. What instruction should the nurse provide?`

A.Come to the clinic today for an ultrasound.

A pregnant client with mitral stenosis Class III is prescribed complete bedrest. The client asks the nurse, "Why must I stay in bed all the time?" Which response is best for the nurse to provide this client? `

A.Complete bedrest decreases oxygen needs and demands on the heart muscle tissue.

The nurse is caring for a woman with a previously diagnosed heart disease who is in the second stage of labor. Which assessment findings are of greatest concern? `

A.Edema, basilar rales, and an irregular pulse.

The nurse attempts to help an unmarried teenager deal with her feelings following a spontaneous abortion at 8-weeks gestation. What type of emotional response should the nurse anticipate? `

A.Grief related to her perceptions about the loss of this child.

Which nursing intervention is helpful in relieving "afterpains" (postpartum uterine contractions)? `

A.Using relaxation breathing techniques.

The nurse is counseling a couple who has sought information about conceiving. For teaching purposes, the nurse should know that ovulation usually occurs `

A.two weeks before menstruation.

The nurse is performing a gestational age assessment on a full-term newborn during the first hour of transition using the Ballard (Dubowitz) scale. Bassed on this assessment the nurse determines that the neonate has a maturity rating 40-weeks. What findings should the nurse identify to determine if the neonate is small for gestational age (SGA)? (select all that apply)

Admission weight of 4 pounds, 15oz Head to heel length of 17in Frontal occipital circumference of 12.5in

Prior to discharge, what instructions should the nurse give to parents regarding the newborn's umbilical cord care at home?

Allow the cord to air-dry as much as possible

The nurse caring for a laboring client encourages her to void at least q2h, and records each time the client empties her bladder. What is the primary reason for implementing this nursing intervention.

An over-distended bladder could be traumatized during labor, as well as prolong the progress of labor.

A client who is having an allergic reaction receives a prescription for epinephrine 0.4 mg subcutaneously. The available vial is labeled, Epinephrine Injection, USP, 1:1000 (1 mg/ml) For Subcutaneous use only. How many ml should the nurse administer? (Enter numerical value only. If rounding is required, round to the nearest tenth.)

Answer 0.4 Rationale

An unlicensed assistive personnel UAP leaves the unit without notifying the staff. In what order should the unit manager implement these interventions to address the UAPs behavior? (Place the actions in order from first on top to last on bottom.)`

Answer 1. Note date and time of the behavior. 2. Discuss the issue privately with the UAP. 3. Plan for scheduled break times. 4. Evaluate the UAP for signs of improvement. Rationale

The nurse provides sliding scale insulin administration instructions to an adult who was recently diagnosed with diabetes mellitus. The client demonstrates an understanding of the instructions provided by performing the procedure in which order? (Arrange with the first on top and the last on the bottom.)`

Answer 1. Obtain blood glucose level. 2. Verify the insulin prescription. 3. Draw insulin into insulin syringe. 4. Cleanse the selected site. Rationale

The nurse is caring for a 4-year-old male child who becomes unresponsive as his heart rate decreases to 40 beats/minute. His blood pressure is 88/70 mmHg, and his oxygen saturation is 70% while receiving 100% oxygen by non-rebreather face mask. In what sequence, from first to last, should the nurse implement these actions? (Place the first action on top and last action on the bottom.)`

Answer 1. Start chest compressions with assisted manual ventilations. 2. Administer epinephrine 0.01 mg/kg intraosseous (IO). 3. Apply pads and prepare for transthoracic pacing. 4. Review the possible underlying causes for bradycardia. Rationale

A client develops urticaria on the trunk and neck shortly after a secondary infusion of piperacillin is initiated. In what order should the nurse implement these interventions? (Arrange the actions in order of priority, with the highest priority first and least priority last or at the bottom.)`

Answer 1. Stop the infusion. 2. Assess vital signs. 3. Contact the healthcare provider. 4. Document reaction to the drug. 5. Initiate adverse event report. Rationale

The nurse is interviewing a 18-year-old female client who was released 3 weeks ago following two months of treatment for anorexia nervosa. Which statement is characteristic of a young woman who has been successfully treated for anorexia nervosa?`

Answer A. "My parents attempt to smother me, but I will not allow them to make my decisions." Rationale

During a family group meeting, the client's daughter tells the group, "I hope I didn't cause mom to be depressed." Which response should the nurse provide?`

Answer A. "You seem worried. What about your mom is bothering you?" Rationale

Four clients arrive on the labor and delivery unit at the same time. Which client should the nurse assess first? `

Answer A. A 39-week primigravida with biophysical profile score of 5 out of 8. Rationale

A 6-year-old who has asthma is demonstrating a prolonged expiratory phase and wheezing, and has 35% of personal best peak expiratory flow rate (PEFR). Based on these finding, which action should the nurse implement first?`

Answer A. Administer a prescribed bronchodilator. Rationale

In making client care assignment, which client is best to assign to the practical nurse (PN) working on the unit with the nurse?`

Answer A. An immobile client receiving low molecular weight heparin q12h. Rationale

The healthcare provider prescribed oxycodone/aspirin 1 tab PO every 4 hours as needed for pain, for a client with polycystic kidney disease. Before administering this medication, which component of the prescription should the nurse question?`

Answer A. Aspirin content. Rationale

The nurse working in a critical care unit is assigned the care of two clients, one with pneumonia who is being mechanically ventilated and the other who had a thoracotomy yesterday and is complaining of incisional pain. What should the nurse to first? `

Answer A. Assess the level of consciousness and vital signs for both clients. Rationale

A postpartum client who is bottlefeeding develops breast engorgement. What is the best recommendation for the nurse to provide this client?`

Answer A. Avoid stimulation of the breasts and wear a tight bra. Rationale

A client who returns from surgery after the removal of a malignant thyroid tumor has a serum calcium level of 4.5 mg/dL or 1.125 mmol-L (SI). Which findings require immediate action by the nurse? (Select all that apply.)`

Answer A. Carpopedal spasms with inflation of the blood pressure cuff. B. Spasm of the cheek and mouth when the facial nerve is tapped. Rationale

The nurse is administering a 750 ml cleansing enema to an adult client. After approximately 150 ml of enema has infused, the client states, "Stop! I can't hold anymore." What action should the nurse take?`

Answer A. Clamp the tubing and instruct the client to breathe deeply before continuing. Rationale

When conducting diet teaching for a client who is diagnosed with Crohn's Disease, which foods should the nurse encourage the client to eat? (Select all that apply.)`

Answer A. Clams. B. Raisins. Rationale

A mother brings her 6-year-old child, who has just stepped on a rusty nail, to the pediatrician's office. Upon inspection, the nurse notes that the nail went through the shoe and pierced the bottom of the child's foot. Which action should the nurse implement first?`

Answer A. Cleanse the foot with soap and water and apply an antibiotic ointment. Rationale

A male client is admitted for the removal of an internal fixation that was inserted for a fracture ankle. During the admission history, he tells the nurse he recently received vancomycin (Vancocin) for a methicillin-resistant Staphylococcus aureus (MRSA) wound infection. Which action should the nurse take? (Select all that apply.)`

Answer A. Collect multiple site screening culture for MRSA. C. Place the client on contact transmission precautions. E. Continue to monitor for client sign of infection. Rationale

During an assessment by the home health nurse of an older man who lives alone, the client reports that he is troubled by constipation. To formulate a plan of care, what additional information should the nurse obtain? (Select all that apply.)`

Answer A. Daily food and fluid intake. B. Current prescribed and over-the-counter medications. D. Level of physical activity and exercise. E. Methods currently used to treat constipation. Rationale

The unlicensed assistive personnel (UAP) reports that a client's blood pressure cannot be measured because the client has casts on both arms and is unable to be turned to the prone position for blood pressure measurement in the legs. What action should the nurse implement?`

Answer A. Demonstrate how to palpate the popliteal pulse with the client supine and the knee flexed. Rationale

The charge nurse observes a new nurse preparing to irrigate an intravenous catheter. The new nurse is attaching a 16 gauge needle. What action should the charge nurse take?`

Answer A. Direct the nurse to remove the needle before the procedure. Rationale

A morbidly obese woman is scheduled for gastric bypass surgery. She completes the required preoperative nutritional counseling and signs the operative permit. To promote effective discharge planning, which intervention is most important for the nurse to implement?`

Answer A. Discuss small, low-fat, low sugar meal preparation techniques. Rationale

The nurse is preparing to send a client to the cardiac cath lab for elective cardioversion. Which intervention should the nurse implement before the client leaves the medical unit?`

Answer A. Document that the client has remained NPO. Rationale

When assessing a client, the nurse should establish which findings as objective? (Select all that apply.)`

Answer A. Edema. D. Diaphoresis. E. Hypertension. F. Urticaria. Rationale

An older adult female asks the clinic nurse about getting a Herpes vaccination because she gets cold sores on her mouth when she is sick or stressed. How should the nurse respond? `

Answer A. Explain the use of the vaccination to reduce risk for Herpes zoster. Rationale

A neonate with a congenital heart defect (CHD) is demonstrating symptoms of heart failure (HF). Which interventions should the nurse include in the infant's plan of care?`

Answer A. Give O2 at 6 L/nasal cannula for 3 repeated oximetry screens below 90%. C. Evaluate heart rate for effectiveness of cardiotonic medications. D. Use high energy formula 30 calories/ounce at q3 hour feeding via soft nipples. E. Ensure uninterrupted and frequent rest periods between procedures. Rationale

The nurse is caring for a client who is taking a macrolide to treat a bacterial infection. Which finding should the nurse report to the healthcare provider before administering the next dose?`

Answer A. Jaundice. Rationale

The client with which type of wound is most likely to need immediate intervention by the nurse?`

Answer A. Laceration. Rationale

An adult male client is admitted to the emergency room following an automobile collision in which he sustained a head injury. What assessment data would provide the earliest indication that the client is experiencing increased intracranial pressure (ICP)?`

Answer A. Lethargy. Rationale

A full-term infant is transferred to the nursery from labor and delicery. Which information is most important for the nurse to receive when planning immediate care for the newborn?

The infants condition at birth and treatment received.

While breastfeeding, a new mother strokes the top of her baby's head and asks the nurse about the baby's swollen scalp. The nurse responds that the swelling is caput succedneum. Which additional information should the nurse provide this new mother?

The scalp edema will subside in a few days after birth

A new mother is afraid to touch her baby's head for fear of hurting the "large soft spot." Which explanation should the nurse give to this anxious client?

There is a strong enough, tough membrane there to protect the baby so you need not be afraid to wash or comb his/her hair

A couple has been trying to conceive for 9 months without success. Which information obtained from the clients is most likely to have an impact on the couple's ability to conceive a child?

They use lubricants with each sexual encounter to decrease friction

A couple has been trying to conceive for nine months without success. Which information obtained from the clients is most likely to have an impact on the couples ability to conceive a child?"

They use lubricants with each sexual encounter to decrease friction.

The nurse is assessing the umbilical cord of a newborn. Which finding constutes a normal finding?

Three vessels: two arteries and one vein

The nurse is counseling a couple who has sought information about conceiving. The couple asks the nurse to explain when ovulation usually occurs. Which statement by the nurse is correct?

Two weeks before menstruation

A 35-year-old primigravida client with severe preeclampsia is receiving magnesium sulfate via continuous IV infusion. Which assessment data would indicate to the nurse that the client is experiencing magnesium sulfate toxicity?

Urine output 90mL / 4 hours

When preparing a class on newborn care for expectant parents, which is correct for the nurse to teach concerning the newborn infant born at term gestation?

Vernix is a white cheesy substance, predominantly located in the skin folds.

The nurse is performing a AGA on a full-term newborn during the first hour of transition using the Dubowitz scale. Based on this assessment, the nurse determines that the neonate has a maturity rating of 40 weeks. Which findings should the nurse identify to determine if the neonate is SGA? (Select all that apply.)`

a, b, c

A full-term infant is admitted to the newborn nursery and, after careful assessment, the nurse suspects that the infant may have an esophageal atresia. Which symptoms is this newborn likely to have exhibited?`

a. Choking, coughing, and cyanosis.

A woman with Type 2 diabetes mellitus becomes pregnant, and her oral hypoglycemic agents are discontinued. Which intervention is most important for the nurse to implement? `

a. Describe diet changes that can improve the management of her diabetes.

On admission to the prenatal clinic, a 23 year old woman tells the nurse that her last menstrual period began on February 15 that previously her periods were regular. Her pregnancy test is positive. This client's expected date of delivery`

a. November 22

A woman who thinks she could be pregnant calls her neighbor, a nurse, to ask wen she could use a home pregnancy test to diagnose pregnancy. Which response is best?

a. a home pregnancy test can be used right after your first missed period

A mutigravida client at 41 weeks gestation present in the labor and delivery unit after a non-stress test indicated that the fetus is experiencing some difficulties in utero. Which diagnostic test should the nurse prepare the client for additional information about fetal status?`

a. biophysical profile

A client at 28 weeks gestation calls the antepartal clinic and states that she is experiencing a small amount of vaginal bleeding which she describes as bright red. She further states that she is not experiencing any uterine contractions or abdominal pain. What instruction should the nurse provide?`

a. come to the clinic today for an ultrasound

Immediately after birth a newborn infant is suctioned, dried, and placed under a radiant warmer. The infant has spontaneous respirations and the nurse assess an apical heart rate of 80 bpm and respirations 20. What action should the nurse perform next?`

a. initiate positive pressure ventilation

A client in active labor is admitted with preeclampsia. Which assessment finding is most significant in planning this client's care?

a. patellar reflex 4+

A client receiving epidural anesthesia begins to experience nausea and becomes pale and clammy. What intervention should the nurse implement first?`

a. raise the foot of the bed

A couple concerned because the woman has not been able to conceive is referred to a HCP for a fertility workup and a hysterosalpingography is scheduled. Which postprocedure complaint indicates that the fallopian tubes are patent?`

a. shoulder pain

The nurse is counseling a couple who has sought information about conceiving. For teaching purposes, the nurses should know that ovulation usually occurs`

a. two weeks before menstruation

Which nursing intervention is helpful in relieving "afterpains"?`

a. using relaxation breathing techniques

The nurse is assessing a 3 day old infant with a cephalohematoma in the newborn nursery. Which assessment finding should the nurse report to the healthcare provider?`

a. yellowish tinge to the skin

A multigravida client at 41-weeks gestation presents in the labor and delivery unit after a non-stress test indicated that the fetus is experiencing some difficulties in utero. Which diagnostic test should the nurse prepare the client for additional information about fetal status? `

a.Biophysical profile (BPP).

When explaining "postpartum blues" to a client who is 1 day postpartum, which symptoms should the nurse include in the teaching plan? (Select all that apply.) `

a.Mood swings. c.Tearfulness

On admission to the prenatal clinic, a 23-year-old woman tells the nurse that her last menstrual period began on February 15, and that previously her periods were regular. Her pregnancy test is positive. This client's expected date of delivery (EDD) is `

a.November 22.

A 42-week gestational client is receiving an intravenous infusion of oxytocin (Pitocin) to augment early labor. The nurse should discontinue the oxytocin infusion for which pattern of contractions? `

a.Transition labor with contractions every 2 minutes, lasting 90 seconds each.

the nurse is assessing a 3-day old infant with a cephalohematoma in the newborn nursery. Which assessment finding should the nurse report to the healthcare provider? `

a.Yellowish tinge to the skin.

When explaining postpartum blues to a client who is 1 day postpartum, which symptoms should the nurse include in the teaching plan? (select all that apply)`

b, d

hich maternal behavior is the nurse most likely to see when a new mother receives her infant for the first time? `

b. Her arms and hands receive the infant and she then traces the infant's profile with her fingertips.

A pregnant client tells the nurse that the first day of her last menstrual period was August 2, 2006. Based on Nagele's rule, what is the estimated date of delivery?`

b. May 9, 2007

A client who has an autosomal dominant inherited disorder is exploring family planning options and the risk of transmission of the disorder to an infant. The nurses's response should be based on what information?`

b. each pregnancy carries 50% chance of inheriting the disorder

A 28 year old client in active labor complains of cramps in her leg. What intervention should the nurse implement?`

b. extend the leg and dorsiflex the foot

A client who is attending antepartum classes asks the nurse why her healthcare provider has prescribed iron tablets. The nurse's response is based on what knowledge?`

b. it is difficult to consume 18 mg of additional iron by diet alone

A 38 week primigravida who works as a secretary and sits at a computer 8 hrs each day tells the nurse that her feet have begun to swell. Which instruction would be most effective in preventing pooling blood in the lower extremities?`

b. move every hour

A new mother who has just had her first baby says to the nurse, "I saw the baby in the recovery room. She sure has a funny looking head." Which response by the nurse is best?`

b. that is normal the head will return to a round shape within 7 to 10 days

A new mother asks the nurse "How do I know that my daughter is getting enough breast milk?" Which explanation should the nurse provide?`

b. your milk is sufficient if the bay is voiding pale straw-colored urine 6 to 10 times a day

A client who has an autosomal dominant inherited disorder is exploring family planning options and the risk of transmission of the disorder to an infant. The nurse's response should be based on what information?`

b.Each pregnancy carries a 50% chance of inheriting the disorder.

A primigravida at 40-weeks gestation is receiving oxytocin (Pitocin) to augment labor. Which adverse effect should the nurse monitor for during the infusion of Pitocin?`

b.Hyperstimulation.

tells the nurse that she want to have an uncomplicated pregnancy and a healthy baby. What information should the nurse share with the client? `

b.Maintain blood sugar levels in a constant range within normal limits during pregnancy.

The nurse observes a new mother avoiding eye contact with her newborn. Which action should the nurse take? `

b.Observe the mother for other attachment behaviors.

The nurse is counseling a woman who wants to become pregnant. The woman tells the nurse that she has a 36-day menstrual cycle and the first day of her last menstrual period was January 8. The nurse correctly calculates that the woman's next fertile period is `

c. January 30-31.

A newborn, whose mother is HIV positive, is scheduled for follow-up assessments. The nurse knows that the most likely presenting symptom for a pediatric client with AIDS is:

c. a persistent cold

A 4 week old premature infant has been receiving epoetin alfa for the last three weeks. Which assessment finding indicates to the nurse that the drug is effective?`

c. changes in apical rate from the 180s to the 140s

A client with no prenatal care arrives at the labor unit screaming, "The baby is coming!" The nurse performs a vaginal examination that reveals the cervix is 3 cm dilated and 75% effaced. What additional information is most important for the nurse to obtain?`

c. date of last normal menstrual period

The total bilirubin level of a 36 hour breastfeeding newborn is 14 mg/dl. Based on this finding which intervention should the nurse implement?`

c. encourage the mother to breastfeed frequently

The nurse instructs a laboring client to use accelerated blow breathing. The client begins to complain of tingling finger and dizziness. What action should the nurse take?`

c. have the client breathe into her cupped hands.

At 14-weeks gestation, a client arrives at the Emergency Center complaining of a dull pain in the right lower quadrant of her abdomen. The nurse obtains a blood sample and initiates an IV. Thirty minutes after admission, the client reports feeling a sharp abdominal pain and a shoulder pain. Assessment findings include diaphoresis, a heart rate of 120 beats/minute, and a blood pressure of 86/48. Which action should the nurse implement next?

c. increase the rate of IV fluids

During a prenatal visit, the nurse discusses with a client the effects of smoking on the fetus. When compared with nonsmokers, mothers who smoke during pregnancy tend to produce infants who have`

c. lower birth weights

The nurse assess a client admitted to the labor and delivery unit and obtains the following data: BP 110/68, FHR 110 bpm, cervix 1 cm dilated and uneffaced. Based on these assessment findings, what intervention should the nurse implement?`

c. monitor for bleeding from IV sites

The nurse observes a new mother avoiding eye contact with her newborn. Which action should the nurse take?`

c. observe the mother for other attachment behaviors

Twenty minutes after a continuous epidural anesthetic is administered, a laboring client's blood pressure drops from 120/80 to 90/60. What action should the nurse take?

c. place the woman in a lateral position

An off-duty finds a woman in a supermarket parking lot delivering an infant while her husband is screaming for someone to help his wife. Which intervention has the highest priority?`

c. put the newborn to breast

A 24-hour-old newborn has a pink papular rash with vesicles superimposed on the thorax, back, and abdomen. What action should the nurse implement?`

c.Document the finding in the infant's record.

The nurse is teaching care of the newborn to a group of prospective parents and describes the need for administering antibiotic ointment into the eyes of the newborn. Which infectious organism will this treatment prevent from harming the infant? `

c.Gonorrhea.

A client is admitted with the diagnosis of total placenta previa. Which finding is most important for the nurse to report to the healthcare provider immediately?`

c.Onset of uterine contractions.

A client who delivered an infant an hour ago tells the nurse that she feels wet underneath her buttock. The nurse notes that both perineal pads are completely saturated and the client is lying in a 6-inch diameter pool of blood. Which action should the nurse implement next? `

c.Palpate the firmness of the fundus

The nurse observes a new mother is rooming-in and caring for her newborn infant. Which observation indicates the need for further teaching?`

c.Places the infant prone in the bassinet.

A 23-year-old client who is receiving Medicaid benefits is pregnant with her first child. Based on knowledge of the statistics related to infant mortality, which plan should the nurse implement with this client? `

c.Teach the client why keeping prenatal care appointments is important.

In developing a teaching plan for expectant parents, the nurse plans to include information about when the parents can expect the infant's fontanels to close. The nurse bases the explanation on knowledge that for the normal newborn, the

d. anterior fontanel closes at 12 to 18 months and the posterior by the end of the second month

A newborn infant is brought to the nursery from the birthing suite. The nurse notices that the infant is breathing satisfactorily but appears dusky. What action should the nurse take first?`

d. check the infant's oxygen saturation rate

A client who delivered an infant an hour ago tells the nurse that she feels wet underneath her buttock. The nurse notes that both perineal pads are completely saturated and the cline is lying in a 6 in diameter pool of blood. Which action should the nurse implement next?`

d. palpate the firmness of the fundus

After each feeding, a 3 day old newborn is spitting up large amounts of newborn formula, a nonfat cow's milk formula. The pediatric healthcare provider changes the neonates's formula to Similac. What information should the nurse provide to the mother about the newly prescribed formula?`

d. similac is a soy based formula that contains sucrose

A woman who had a miscarriage 6 months ago becomes pregnant. Which instruction is most important for the nurse to provide this client?`

d. take prescribed multivitamin and mineral supplements

A primigravida client who is 5 cm dilated, 90% effaced, and at 0 station is requesting an epidural for pain relief. Which assessment finding is most important for the nurse to report to the healthcare provider? `

d.A platelet count of 67,000/mm3.

Which assessment finding should the nursery nurse report to the pediatric healthcare provider? `

d.Central cyanosis when crying.

What action should the nurse implement to decrease the client's risk for hemorrhage after a cesarean section? `

d.Check the firmness of the uterus every 15 minutes.

the nurse is planning preconception care for a new female client. Which information should the nurse provide the client?`

d.Encourage healthy lifestyles for families desiring pregnancy.

A client who is in the second trimester of pregnancy tells the nurse that she wants to use herbal therapy. Which response is best for the nurse to provide?`

d.It is important that you want to take part in your care.

The nurse is calculating the estimated date of confinement (EDC) using Nägele's rule for a client whose last menstrual period started on December 1. Which date is most accurate? `

d.September 8.

While breastfeeding, a new mother strokes the top of her baby's head and asks the nurse about the baby's swollen scalp. The nurse responds that the swelling is caput succedaneum. Which additional information should the nurse provide this new mother? `

d.The scalp edema will subside in a few days after birth.


Related study sets

Government Chapter 3 Test Review

View Set

Kinesiology Exam 3 Multiple Choice Questions (Final)

View Set

MID TERM BUSINESS PRINCIPLES Chapter 1

View Set

CHAPTER 4: ECONOMIC CHARACTERISTICS OF AIRLINES

View Set

Chapter 1: Hierarchical Network Design

View Set